1 класс

Задачи по математике 11 класс с ответами и решением: Национальный исследовательский университет «Высшая школа экономики»

Содержание

Математика | Олимпиадные задания

Содержание Задача 6.1 Задача 6.2 Задача 6.3 Задача 6.4 Задача 6.5 Задача 6.6 Задача 6.7 Задача 6.8 Задача 6.1 Найдите любое решение ребуса где A, B, C — три различные ненулевые цифры; запись AB означает двузначное число, составленное из цифр A и B; запись CCC означает трёхзначное число, состоящее только из цифр C. В качестве […]

 
8189

 
30.11.2020

Содержание Задача 5.1 Задача 5.2 Задача 5.3 Задача 5.4 Задача 5.5 Задача 5.6 Задача 5.7 Задача 5.8 Задача 5.1. Денис расставил числа от 1 до 9 в клетки квадрата 3 × 3 так, что сумма чисел во всех строках и во всех столбцах равна 15. А Лёша стёр числа от 1 до 5 и вместо […]

 
9719

 
24.11.2020

Содержание Задача 4.1 Задача 4.2 Задача 4.3 Задача 4.4 Задача 4.5 Задача 4.6 Задача 4.7 Задача 4.8 Задача 4.1 Винни-Пух и Пятачок договорились утром пойти в гости к Кролику. Пятачок встал пораньше и решил сначала дойти до домика Винни-Пуха, а потом вместе с другом к полудню быть у Кролика. Какое расстояние прошёл Пятачок до полудня, […]

 
8382

 
17.11.2020

Содержание Задача 3.1 Задача 3.2 Задача 3.3 Задача 3.4 Задача 3.5 Задача 3.6 Задача 3.7 Задача 3.8 Задача 3.1 На доске было написано четыре арифметических примера. Маша стёрла числа 1, 2, 3, 4, 5 и написала вместо них буквы (a) Вместо буквы A (1) стояло число 1. (b) Вместо буквы B (2) стояло число 2. […]

 
9202

 
03.11.2020

Содержание Задача 1 Задача 2 Задача 3 Задача 4 Задача 5 Задача 6 Задача 1 Докажите, что уравнение x2 + 22018x + 22019 = 0 не имеет целых корней. Первое решение. Дискриминант этого уравнения равен 24036 — 4 · 22019 = 22021(22015 — 1). Для наличия целого корня необходимо, чтобы дискриминант был точным квадратом. Однако, […]

 
10194

 
02.10.2019

Содержание Задача 1 Задача 2 Задача 3 Задача 4 Задача 5 Задача 6 Задача 1 Разрежьте правильный пятиугольник на пять равных треугольников и один правильный пятиугольник меньшего размера. Решение. Проведём лучи из вершин пятиугольника под равными углами со смежными с ними сторонами, как на рисунке. Каждый луч доведём до пересечения со следующим лучом. Критерии 0 б. […]

 
12238

 
01.10.2019

Содержание Задача 1 Задача 2 Задача 3 Задача 4 Задача 5 Задача 6 Задача 1 Есть три брата-акробата. Их средний рост — 1 метр 74 сантиметра. А средний рост двух из этих братьев: самого высокого и самого низкого — 1 метр 75 сантиметров. Какого роста средний брат? Ответ обоснуйте. Ответ: 1 метр 72 сантиметра. Решение. […]

 
16682

 
01.10.2019

Содержание Задача 1 Задача 2 Задача 3 Задача 4 Задача 5 Задача 6 Задача 1 В числовом выражении некоторые цифры заменили буквами (разные цифры — разными буквами, одинаковые цифры — одинаковыми буквами). Получилось следующее: 2018A : BCD = AA : Какое числовое выражение было записано изначально? (Достаточно привести пример. 2018A изначально было пятизначным числом.) Ответ: […]

 
20969

 
30.09.2019

Содержание Задача 1 Задача 2 Задача 3 Задача 4 Задача 5 Задача 1 Антон выписал на доску арифметическое выражение, а Лёня заменил в нём некоторые цифры буквами (разные цифры — разными буквами, одинаковые цифры — одинаковыми буквами). Получилось следующее: Восстановите выражение. (Достаточно привести пример.) Ответ: Замечание. Других решений не существует. Критерии 4 б. Приведён верный […]

 
24914

 
30.09.2019

Содержание Задача 1 Задача 2 Задача 3 Задача 4 Задача 5 Задача 1 Лёня умеет умножать числа на 7, Глеб — прибавлять 3, Саша — делить на 4, Андрей — вычитать 5. В каком порядке им нужно выполнять свои операции (каждую ровно 1 раз), чтобы получить из числа 8 число 30? Ответ: (8 : 4 […]

 
28651

 
27.09.2019

Содержание Задача 1 Задача 2 Задача 3 Задача 4 Задача 5 Задача 1 Впишите в квадратики числа от 1 до 5, чтобы получилось верное равенство (каждое число используется ровно один раз): Достаточно привести один пример. Ответ: 1 + 2 = 3 · (5 — 4). Замечание. Возможны и другие примеры. Критерии 4 б. Приведён верный […]

 
33151

 
27.09.2019

Содержание Задача 1 Задача 2 Задача 3 Задача 4 Задача 5 Задача 1 На доске в ряд выписаны цифры 1 1 1 2 2 2 5 5 5 : Между ними можно расставить несколько плюсов, чтобы получившийся результат заканчивался на цифру один: 1 + 1 + 12 + 2 + 25 + 5 + 5 […]

 
17325

 
26.09.2019

Содержание Задание 1 Задание 2 Задание 3 Задание 4 Задание 5 Задание 6 Задание 1 Содержание ↑ Графики функций y = ax2 , y = bx и y = c пересекаются в точке, расположенной выше оси абсцисс. Определите, сколько корней может иметь уравнение ax2 + bx + c = 0 . Ответ: корней нет. Решение. Из […]

 
3847

 
03.02.2018

Содержание Задание 1 Задание 2 Задание 3 Задание 4 Задание 5 Задание 6 Задание 1 Содержание ↑ 33 богатыря выходят в дозор 33 дня. В первый день должен выйти один богатырь, во второй – два, в третий – три, и так далее, в последний день – все богатыри. Сможет ли дядька Черномор организовать дозоры так, чтобы все […]

 
4321

 
03.02.2018

Содержание Задание 1 Задание 2 Задание 3 Задание 4 Задание 5 Задание 6 Задание 1 Содержание ↑ Игорь сложил десять подряд идущих натуральных чисел, затем разделил полученную сумму на сумму следующих десяти натуральных чисел. Могло ли у него получится число 0,8? Ответ: не могло. Решение. Предположим, что при делении получилось 0,8. Обозначим наименьшее число первой суммы […]

 
5788

 
03.02.2018

Демонстрационные варианты (демоверсии) ЕГЭ по математике

Демонстрационные варианты ЕГЭ по математике для 11 класса за 2002-2009 годы включали в себя три раздела: А (задачи с выбором ответа из нескольких предложенных), В (задачи с кратким ответом) и С (задания, для выполнения которых требовалось привести полное решение задачи).

В 2010 году из демонстрационного варианта ЕГЭ по математике были исключены задачи с выбором ответа, ранее составлявшие раздел А. Таким образом, демонстрационный вариант ЕГЭ стал состоять уже только из двух разделов В и С.

Демонстрационный вариант ЕГЭ 2011 года почти полностью совпадал с демонстрационным вариантом ЕГЭ 2010 года: были изменены лишь задания C1 и C5.

В 2014 году в демонстрационном варианте ЕГЭ по математике тематических изменений по сравнению с предыдущим годом не было: задачи В3, В9, В14, С2 и С4 были заменены на другие задачи той же тематики. Кроме того, было добавлено задание базового уровня сложности с кратким ответом, проверяющее практические навыки применения математики в повседневной жизни и изменен порядок заданий.

В 2015 году в порядке проведения ЕГЭ по математике произошли серьезные изменения: было решено проводить два отдельных экзаменабазового уровня и профильного уровня.

В связи с этим в 2015 году было представлено 2 демонстрационных варианта: новая модель демонстрационного варианта для ЕГЭ базового уровня и модернизированная модель демонстрационного варианта 2014 года для проведения ЕГЭ профильного уровня.

Демонстрационный вариант для ЕГЭ базового уровня содержал только задания базового уровня сложности с кратким ответом (20 заданий). В демонстрационном варианте было представлено по несколько примеров заданий на каждую позицию экзаменационной работы. В реальных вариантах экзаменационной работы на каждую позицию было предложено только одно задание.

Демонстрационный вариант профильного экзамена 2015 года разработан на основе демонстрационного варианта ЕГЭ по математике 2014 года со следующими изменениями:

  • Вариант стал состоять из двух частей (часть 1 — задания с кратким ответом, часть 2 — задания с кратким ответом и задания с развернутым ответом).
  • Нумерация заданий стала сквозной по всему варианту без буквенных обозначений В, С.
  • Во второй части добавлено 1 задание высокого уровня сложности с развёрнутым ответом, проверяющее практические навыки применения математики в повседневной жизни, навыки построения и исследования математических моделей.
  • Из первой части исключено 1 задание базового уровня сложности.
  • Произведены несущественные изменения формы и тематики заданий 16 и 17

В демонстрационном варианте ЕГЭ по математике базового уровня 2016 года изменений не было .

В демонстрационном варианте ЕГЭ по математике профильного уровня 2016 года произошли следующие изменения:

  • Из первой части варианта были исключены два задания: задание практического содержания базового уровня сложности и задание по стереометрии повышенного уровня сложности.
  • Максимальный первичный балл за выполнение всей работы был уменьшен с 34 до 32 баллов.

В демонстрационных вариантах ЕГЭ по математике 2017 — 2021 годов как базового уровня, так и профильного уровня, по сравнению с демонстрационными вариантами ЕГЭ по математике 2016 года изменений не было.

Олимпиадные задания по математике 10-11 класс


Просмотр содержимого документа

«10 и 11 классы »


Просмотр содержимого документа

«10 класс решение»

10 класс

Решения

  1. Решение.

, т.к. первое слагаемое – это произведение трех последовательных натуральных чисел, т.е. оно кратно 3, а второе слагаемое содержит множитель 3, значит и вся сумма кратна 3.

  1. Решение.

Проведем BK параллельно CD. Заметим KD || BC, KB || DC, следовательно, KBCD параллелограмм и KD = BC =. AD – секущая параллельных прямых BK и CD, следовательно AKB =ADC = 30°.

Далее найдем длину отрезка AK = ADKD = . Боковую сторону AB теперь можно найти по теореме синусов для треугольника ABK: . При этом ABK = 180° – AKB – BKA = 180° – 30°– 15° = 135°.

И sin 135° = . Теперь можно найти AB, она получается равной 1.

Ответ: 1.

  1. Ответ: на семь слагаемых.

Решение. Приведём пример разбиения числа 96 на семь слагаемых:

9 6 = 2 + 5 + 7 + 11 + 13 + 17 + 41.

Если слагаемых больше, то среди них не менее восьми нечётных (если их семь, то сумма нечётна). Заменим каждое из них на наименьший простой сомножитель. При этом сумма не увеличится, и все слагаемые будут различны. Но сумма восьми наименьших нечётных простых чисел равна 98.

  1. Решение.

Имя

Белое платье

Красное платье

Голубое платье

Туфли

Тамара

+

Красные

Лида

+

Голубые

Валя

+

Белые

Ответ: у Тамары были красные туфли и платье, у Вали – белые туфли и голубое платье, у Лиды – белое платье и голубые туфли.

  1. Ответ: х = 95, у = 0, z = 94 или х = 31, у = 2, z = 32.

Решение. Вычтя из второго уравнения первое, получим (х — z)(1 — у) = 1.

По условию, х, у, z целые, тогда возможны два случая:

1) х– z = 1, 1 – у = 1, т. е. у = 0. Подставив значение у в систему, получим: z =94, x=95.

2) х –z = -1, 1 – у = — 1, т. е. z = х +1, у = 2. Подставим найденные значения у и z в первое уравнение, получим 2х + х +1 = 94, х = 31. Отсюда z = 32 .


Просмотр содержимого документа

«11 класс решение»

11 класс

Решение

  1. Ответ. в 17.00.

Решение. Расстояние между Мишей и Колей и их скорости не меняются, а скорости Васи и Пети равны. Вася встретил Колю через 2 часа после Миши, значит, Петя встретят Колю тоже через 2 часа после Миши, т. е. в 17.00.

  1. Ответ. 17.

Решение. Так как из 18 шаров найдется хотя бы один синий, то красных не более 17, а из любых 10 шаров найдется хотя бы один красный, то есть синих не более 9. Так как всех шаров 26, то синих – 9, а красных – 17.

  1. Ответ. 70 кг

Решение. 5 процентов от 30 кг — = 1,5 кг соли в 30кг морской воды,

Х л добавили, стало (х+30)л. Х+30-100 процентов, 1,5 -1,5 процентов, тогда х=70

  1. Ответ: , k꞊1+5t, t.

Решение. tgx ctgx ꞊ sin5x

, k, n

Найдем теперь такие k, при которых ꞊. Это уравнение перепишем в виде 5n-4k꞊1.Частное решение последнего уравнения

n0꞊1, k0꞊1,тогда k꞊1+5t, t. Итак, k, t

  1. Ответ:

Решение. Обозначим длину отрезка AB за 1, тогда SA=2. Найдём прежде всего длины отрезков BD и SD. Пусть BD=x. Тогда, применяя теорему Пифагора к треугольникам ABD и ASD, получаем AD²=1–x² = 4 – (2–x)², BD=x=1/2. Далее в треугольнике BMD BM꞊и, для того чтобы воспользоваться теоремой косинусов, достаточно найти косинус угла MBD. Но из прямоугольного треугольника SBM получаем: cos SBM ꞊

DM ² ꞊꞊, DM ꞊

ЕГЭ по математике, подготовка к ЕГЭ по математике 2021 в Москве, шкала перевода баллов — Учёба.ру

Что требуется

Решить планиметрическую задачу.

Особенности

Под этим номером может быть два варианта задания. Первый вариант: в задаче два пункта — а и b. В пункте a требуется что-то доказать, в пункте b — что-то найти. Могу сказать, что чаще всего надо начинать решать эту задачу именно с пункта b, а уже решение этого пункта поможет доказать пункт а. Как правило, абитуриентам проще что-то найти, чем доказать.

Второй вариант: задача без подпунктов. Здесь чаще всего скрыт подводный камень: задача требует рассмотрения двух случаев и приводит к двум разным ответам. Например, в условии задачи сказано, что окружности касаются в точке A, но не сказано каким образом, внешним или внутренним. Часто бывает так, что выпускник рисует один рисунок и возможно даже находит правильный ответ. А второй случай он не рассматривает, в результате чего получает ровно половину баллов за это задание.

Советы

Необходимое условие для решения этой задачи — хорошее владение теоретическим материалом, например, из классического учебника по геометрии для 7-9 классов (Л.С. Атанасян). Необходимо знать формулировки аксиом и определений, уметь формулировать и доказывать теоремы, признаки, свойства и формулы. Изучите дополнительные методы: метод дополнительного построения, метод подобия, метод замены, метод введения вспомогательного неизвестного, метод удвоения медианы, метод вспомогательной окружности, метод площадей.

Также здесь важен рисунок. 80% успеха геометрической задачи — это правильно нарисованный рисунок. Сделайте большой, хороший, наглядный рисунок, не экономьте на нем место.

И последнее, лайфхак для абитуриента — для решения задач по планиметрии выучите пять формул площади треугольника: через высоту и основание, через две стороны и угол между ними, через радиус вписанной окружности, через радиус описанной окружности и формулу Герона.

Всероссийская олимпиада по математике «Отличник»

Олимпиада по математике «Отличник» 2021 — это всероссийская дистанционная олимпиада по математике для школьников 1-11 классов и студентов первых курсов учреждений среднего профессионального образования.

Участие в олимпиаде по математике — это замечательная возможность углубленного изучения такого важного школьного предмета как математика.

Олимпиада по математике предоставляет возможность решать сложные, нестандартные, но в то же время доступные задания, существенно повышает уровень подготовки и создаёт хорошую базу для дальнейшей учёбы в школе или вузе.

Всероссийский конкурс по математике «Отличник» — это отличный шанс для школьников проявить себя, раскрыть математические способности, подготовиться к другим конкурсам по математике, приобрести уверенность в себе и своих силах.

 

Награды участникам олимпиады

В зависимости от того, как решены задания, каждому участнику дистанционной олимпиады по математике вручается Сертификат или Диплом победителя 1, 2 или 3 степени.

Всем педагогам и координаторам вручаются «Благодарственные письма» за помощь в организации олимпиад и конкурсов.

Если же ученики получат Диплом 1, 2 или 3 степени, то учитель получает «Свидетельство о подготовке победителя».

Скачать дипломы и свидетельства можно  сразу  после ввода ответов.

 

 

Задания конкурса

Математическая олимпиада «Отличник» представляет собой набор из 10 заданий школьной программы и олимпиадных задач различной сложности:

  • 3 простых задания (3 балла)
  • 4 средних задания (5 баллов)
  • 3 сложных задания (7 баллов)

За каждое задание начисляются баллы (3, 5 и 7 соответственно).

Максимально участник может набрать 50 баллов.

Рекомендуемое время на выполнение всех заданий олимпиады: 1 час.

Примеры заданий по математике

 

Работа над ошибками

После завершения конкурса участнику дается возможность проделать работу над ошибками.

К заданиям на конкурс по математике есть ответы с решением и пояснениями важных и ключевых математических нюансов.

При возникновении вопросов, Вы всегда можете обратиться за разъяснениями к нашим специалистам.

 

 

Математика вокруг нас

Друзья, оглянитесь! Вокруг нас появляется столько новых технологий и изобретений, просто невозможных без математики; навыки вычислений, умение правильно считать требует от Вас каждая хорошая профессия, не говоря уже о просто походе за покупками.

Математика – «царица наук», и это не случайно – она существует во всем.

В наше время у нас есть отличная возможность учиться и развиваться каждый день на протяжении всей жизни, поэтому математические навыки и умения улучшать и преумножать никогда не поздно!

Основоположник современной механики и физики Галилео Галилей говорил:

«Математика — это язык, на котором написана книга природы».

От познания этой великой науки можно получить неимоверное удовольствие.

Математический конкурс, безусловно, очень полезен для всех школьников, в нем отрабатывается безукоризненный подход к пониманию механики окружающего мира, улучшается логическое мышление и способность действовать, четко анализируя ситуацию. Улучшение памяти при этом является закономерным приятным последствием.

 

Цели и задачи мероприятия

  • углубленное изчение предмета;
  • активизация абстрактного и логического мышления;
  • проверка уровня знаний и умений;
  • совершенствование способности ребят применять знания, полученные на уроках;
  • повышение интереса к изучению предмета;
  • определение самых активных и способных учащихся;
  • награждение учащихся за стремление к изучению предмета.

 

Темы для подготовки к олимпиаде

Для участников разных возрастных групп (классов) предусмотрены соответствующие наборы заданий олимпиады, которые могут включать в себя задачи на следующие темы. Используйте их для подготовки и успешного решения заданий.

Олимпиада по математике 1-2 класс

  • Сложение и вычитание, счет предметов
  • Элементы комбинаторики для начальной школы
  • Продолжение числового ряда
  • Задачи с числами, решение числовых ребусов
  • Нахождение неизвестного компонента

Олимпиада по математике 3 класс

  • Использование основных арифметических действий
  • Нахождение периметра фигуры
  • Решение числового ребуса
  • Натуральные числа и десятичная запись числа
  • Продолжение числового ряда
  • Задачи с числами
  • Элементы комбинаторики для начальной школы

Олимпиада по математике 4 класс

  • Задачи на движение
  • Развитие навыков использования частей числа
  • Знание единиц измерения
  • Умножение и деление, сложение и вычитание
  • Решение числового ребуса
  • Числа, подсчет количества фигур

Олимпиада по математике 5 класс

Олимпиада по математике 6 класс

  • Делимость натуральных чисел и признаки делимости
  • Сложение и вычитание дробей с разными знаменателями
  • Умножение и деление дробей
  • Отношения и пропорции
  • Положительные и отрицательные числа
  • Сложение и вычитание положительных и отрицательных чисел
  • Умножение и деление положительных и отрицательных чисел
  • Решение уравнений
  • Координаты на плоскости

Олимпиада по математике 7 класс

  • Математический язык и математическая модель
  • Линейная функция. График линейной функции.
  • Системы линейных уравнений
  • Одночлены. Арифметические операции над одночленами.
  • Многочлены. Арифметические операции над многочленами.
  • Разложение многочлена на множители
  • Функция y = x2
  • Начальные геометрические сведения
  • Треугольники
  • Параллельные прямые
  • Соотношения между сторонами и углами треугольника

Олимпиада по математике 8 класс

  • Алгебраические дроби
  • Функция y =  √x . Свойства квадратного корня.
  • Квадратичная функция
  • Функция y = k/x
  • Квадратные уравнения
  • Неравенства
  • Четырехугольники
  • Площадь
  • Подобные треугольники
  • Окружность

 

Олимпиада по математике 9-11 класс и 1-2 курс СПО

  • Задания с числами
  • Уравнения, содержащее квадратные корни
  • Нахождение области определения функций
  • Геометрические задачи
  • Текстовые задачи на смеси и сплавы
  • Элементы теории вероятности
  • Решение тригонометрических уравнений

 

Олимпиада по математике – это важно

Термин «олимпиада» пришел к нам из Древней Греции, но в наше время приобрел новое значение, а именно трансформировался в такое понятие, как «олимпиада по математике». Такой вид конкурса умов и интеллекта становится с каждым годом все популярнее в кругу школьников.

Олимпиадные задания каждый год становятся интереснее и доступнее с появлением дистанционной формы участия. Школьники оттачивают навыки запоминания огромного количества информации, активируется скрытые способности мозга человека, ведь конкурсы по математики направлены именно на логическое мышление и использует непростые навыки вычисления и анализа.

 

Конкурс по математике — это перспективно

В наше время люди, умеющие мыслить быстро и четко, умеющие находить ответы даже в самых непредсказуемых ситуациях, востребованы как никогда. Участвуя в олимпиадах и конкурсах по математике, вы не только тренируетесь в решении задач, но укрепляете свою уверенность в успехе.

Это хорошая подготовка к региональным и муниципальным очным мероприятиям по математике, заняв призовое место в которых, можно очень качественно пополнить портфолио долгожданной наградой.

 

Диплом победителя олимпиады по математике

Согласитесь, что получить Диплом Победителя олимпиады по математике – это очень престижно и приятно? Всероссийская олимпиада по математике «Отличник» предоставляют каждому замечательную возможность осуществить свою давнюю мечту и стать призером.

Онлайн конкурс по математике «Отличник» позволят Вам быстро и удобно принять участие в конкурсе по математике и проявить себя, показать математические навыки, открыть в себе настоящего математика!

 

Желаем удачи и отличных результатов!

Решение математических задач с физическим содержанием, 11 класс

Интегрированный урок-семинар

(математика + физика)

в 11 классе по теме:

«Решение задач с физическим

содержанием»

Составили : учитель математики

МОУ «СОШ № 45»

Фазлиахметова Н. В.,

высшая категория, и

учитель математики

МОУ «СОШ № 80»

Крылёва Л.М.,

высшая категория.

Кемерово 2010г.

Интегрированный урок

(математика + физика) в 11 классе.

«Решение задач с физическим содержанием».

Цель:

физической ситуации.

  • Расширение круга задач, решаемых с помощью алгебраических методов.

  • Развитие познавательного интереса учащихся, умение работать с дополнительной литературой.

  • Воспитание ответственности и самостоятельности при подготовке к семинару.

Задачи урока:

Образовательные:

Проверка умений учащихся решать задачи с физическим содержанием

Формирование умений устанавливать отношения между предметами с

помощью прикладных программ.

Применение полученных знаний на практике.

Развивающие:

Развитие логического мышления, умения делать выводы.

Развитие умения применять информационные технологии для оформления работ и решения задач с современными требованиями.

Воспитательные:

Воспитание информационной культуры.

Стимулирование познавательной деятельности постановкой

проблемных вопросов и заданий

Воспитание умения работать в группе.

Планируемые результаты:

Знать:

Физические формулы, используемые при решении математических задач.

Решение неравенств второй степени.

Уметь:

Применять эти формулы на практике.

Решать задачи физического содержания.

ТИП УРОКА: интегрированный урок-семинар решения задач с физическим содержанием для учащихся 11 классов (2 часа).

Комплексно- методическое обеспечение: интерактивная доска, таблицы с формулами, плакаты с высказываниями, выставка книг.

Методы обучения:

  • Объяснительно- иллюстративный

  • Репродуктивный

  • Частично-поисковый

  • Проблемный

План проведения урока- семинара.

  1. Вступительное слово учителя математики.

  2. Выступление учащихся от каждой группы.

  3. Самостоятельная работа.

  4. Подведение итогов урока- семинара.

Организация урока- семинара.

Класс разбивается на 6 групп. Каждая группа получает задание разобрать и решить

определённую группу физических задач. Учащиеся при подготовке к семинару

прорабатывает соответствующие разделы учебников, использует интернет,

дополнительную литературу, получает консультацию учителей физики и

математики. На подготовку к уроку отводится неделя.

ХОД УРОКА-СЕМИНАРА.

1. Организационный момент

Учителем сообщается тема урока, цель его проведения.

Эпиграфом к сегодняшнему уроку послужат следующие слова:

«Образование есть то, что остаётся у человека,

когда остальное забывается»

2. Систематизация знаний

Вступительное слово учителя:

Задания с прикладным содержанием, включённые в 2010 году в экзаменационные

варианты ЕГЭ по математике под номером В10 представляют собой достаточно

широкий круг: это и задачи с экономическим содержанием, и задачи о тепловом

расширении тел, о сокращении длины быстро движущихся ракет, об определении глубин

колодцев и об исследовании температуры звёзд, о проектировании подводных

аппаратов, о скейтбордистах и даже о водолазных колоколах. Научиться решать

задачи – одна из важнейших целей образования. Овладеть математическими знаниями,

позволяющими описывать окружающий нас мир, научиться составлять, анализировать и

интерпретировать соответствующие математические модели – наиважнейшая цель

математического образования. Помочь хотя бы немного в этом нелёгком труде и

призван наш сегодняшний урок.

Представитель каждой группы рассказывает остальным учащимся о задачах, над которыми работала его группа. Один ученик объясняет физический смысл задачи и строит математическую модель данной физической ситуации. Другой ученик показывает решение задачи уже алгебраическим методом.

1 группа

Задача № 1.При температуре рельс имеет длину =12,5 м. При возрастании температуры происходит тепловое расширение рельса, и его длина , выраженная в метрах, меняется по закону , где коэффициент теплового расширения , температура (в градусах Цельсия). При какой температуре рельс удлинится на 6 мм? Ответ выразите в градусах Цельсия.

Решение.

Задача сводится к решению уравнения = 6 (мм) при заданных значениях длины =12,5 м и коэффициента теплового расширения =1,2 :

= 6 (м) ⇔ = 6

⇔ 12,5(1+1,2)12,5 = 6

⇔ 12,512,5 = 6

= 6

40.

Ответ: 40.

2 группа

Задача № 2.В боковой стенке высокого цилиндрического бака у самого дна закреплен кран. После его открытия вода начинает вытекать из бака , при этом высота столба воды в нем, выраженная в метрах, меняется по закону + , где

время в секундах, прошедшее с момента открытия крана, 20 мначальная высота столба воды, отношение площадей поперечных сечений крана и бака,

а ускорение свободного падения ( считайте g 10 м/ ). Через сколько секунд после

открытия крана в баке останется четверть первоначального объёма воды?

Решение.

Задача сводится к решению уравнения при заданных значениях начальной

высоты , отношения площадей поперечных сечений крана и бака и

ускорения свободного падения g 10 м/ :

Решив квадратное уравнение, имеем и .

не удовлетворяет условию задачи.

Ответ: 400 с.

3 группа

Задача № 3.Для определения эффективной температуры звезд используют закон Стефана-Больцмана, согласно которому мощность излучения нагретого тела , измеряемая в ваттах, прямо пропорциональна площади его поверхности и четвертой степени температуры :, где 5,7постоянная, площадь измеряется в квадратных метрах, температура в градусах Кельвина, а мощность в ваттах. Известно, что некоторая звезда имеет площадь поверхности , а излучаемая ею мощность не менее 2,28 Вт. Определите наименьшую возможную температуру этой звезды. Приведите ответ в градусах Кельвина.

Решение.

Задача сводится к решению неравенства при известном значении постоянной 5,7 и заданной площади поверхности звезды :

⇔ 5,7

4000 K

Значит, наименьшая возможная температура звезды 4000 K.

Ответ: 4000 К.

4 группа

Задача № 4.Для получения на экране увеличенного изображения лампочки в лаборатории используется собирающая линза с главным фокусным расстоянием см. Расстояние от линзы до лампочки может изменяться в пределах от 35 см до 60 см, а расстояние от линзы до экрана в пределах от 240 см до 280 см. Изображение на экране будет четким, если выполнено соотношение Укажите, на каком наименьшем расстоянии от линзы можно поместить лампочку, чтобы её изображение на экране было четким. Ответ выразите в сантиметрах.

Решение.

Поскольку 35 см, имеем :⇔ : .

Наименьшему возможному значению соответствует наибольшее значение левой части полученного равенства, и, соответственно, наибольшее возможное значение правой части равенства. Разность в правой части равенства достигает наибольшего значения при наименьшем значении вычитаемого , которое достигается при наибольшем возможном значении знаменателя .

Поэтому =280, откуда см.

По условию лампочка должна находиться на расстоянии от 35 см до 60 см от линзы. Найденное значение см удовлетворяет условию.

Ответ: 40 см.

5 группа

Задача № 5.При движении ракеты её видимая для неподвижного наблюдателя длина, измеряемая в метрах, сокращается по закону , где 5 мдлина покоящейся ракеты, км/с скорость света, а скорость ракеты (в км/с ). Какова должна быть минимальная скорость ракеты, чтобы её наблюдаемая длина стала не более 3 м ? Ответ выразите в км/с.

Решение.

Найдем, при какой скорости длина ракеты станет равна 3 м. Задача сводится к решению уравнения = 3 (м) при заданном значении длины покоящейся ракеты 5 м и известном значении скорости света км/с:

= 3 (км) ⇔ = 3

=

=

= 2,4

240000 км/с.

Если скорость будет превосходить найденную, то длина ракеты будет менее 3 м, поэтому минимальная необходимая скорость будет равна 240000 км/с.

Ответ: 240000 км/с.

6 группа

Задача № 6.Находящийся в воде водолазный колокол, содержащий моля воздуха при давлении атмосферы, медленно опускают на дно водоёма. При этом происходит изотермическое сжатие воздуха. Работа (в джоулях), совершаемая водой при сжатии воздуха, определяется выражением ,

где = 5,75постоянная, 300 K температура воздуха, (в атм)начальное давление, а (в атм)конечное давление воздуха в колоколе. До какого наибольшего давления (в атм) можно сжать воздух в колоколе, если при сжатии воздуха совершается работа не более , чем 6900 Дж?

Решение.

Задача сводится к решению неравенства 6900 при заданных значениях количества воздуха моля, его начального давления атмосферы и температуры 300 K, а также постоянной = 5,75:

6900 ⇔ 6900 ⇔ 5,75 6900 ⇔

2 .

Ответ: 6 атмосфер.

После выступления представителей от каждой группы, обсуждается решение задачи, задаются вопросы и учитель подводит итог по решению данного вида задач.

3.Самостоятельная работа (15 минут)

I вариант

Задача № 1.При температуре рельс имеет длину =15 м. При возрастании температуры происходит тепловое расширение рельса, и его длина, выраженная в метрах, меняется по закону , где коэффициент теплового расширения , температура (в градусах Цельсия). При какой температуре рельс удлинится на 4,5 мм? Ответ выразите в градусах Цельсия.

Задача № 2.В боковой стенке высокого цилиндрического бака у самого дна закреплен кран. После его открытия вода начинает вытекать из бака , при этом высота столба воды в нем, выраженная в метрах, меняется по закону + , где

время в секундах, прошедшее с момента открытия крана, 5 мначальная высота столба воды, отношение площадей поперечных сечений крана и бака,

а ускорение свободного падения ( считайте g 10 м/ ). Через сколько секунд после

открытия крана в баке останется четверть первоначального объёма воды?

II вариант

Задача № 1.Для определения эффективной температуры звезд используют закон Стефана-Больцмана, согласно которому мощность излучения нагретого тела , измеряемая в ваттах, прямо пропорциональна площади его поверхности и четвертой степени температуры :, где 5,7постоянная, площадь измеряется в квадратных метрах, температура в градусах Кельвина, а мощность в ваттах. Известно, что некоторая звезда имеет площадь поверхности , а излучаемая ею мощность не менее 2,85 Вт. Определите наименьшую возможную температуру этой звезды. Приведите ответ в градусах Кельвина.

Задача № 2.Для получения на экране увеличенного изображения лампочки в лаборатории используется собирающая линза с главным фокусным расстоянием см. Расстояние от линзы до лампочки может изменяться в пределах от 50 см до 70 см, а расстояние от линзы до экрана в пределах от 160 см до 180 см. Изображение на экране будет четким, если выполнено соотношение Укажите, на каком наименьшем расстоянии от линзы можно поместить лампочку, чтобы её изображение на экране было четким. Ответ выразите в сантиметрах.

III вариант

Задача № 1.При движении ракеты её видимая для неподвижного наблюдателя длина, измеряемая в метрах, сокращается по закону , где 25 мдлина покоящейся ракеты, км/с скорость света, а скорость ракеты (в км/с). Какова должна быть минимальная скорость ракеты , чтобы её наблюдаемая длина стала не более 7 м ? Ответ выразите в км/с.

Задача № 2.Находящийся в воде водолазный колокол, содержащий моля воздуха при давлении атмосферы, медленно опускают на дно водоёма. При этом происходит изотермическое сжатие воздуха. Работа ( в джоулях ), совершаемая водой при сжатии воздуха, определяется выражением , где = 9,15постоянная, 300 K температура воздуха, (в атм)начальное давление, а (в атм)конечное давление воздуха в колоколе. До какого наибольшего давления (в атм) можно сжать воздух в колоколе, если при сжатии воздуха совершается работа не более , чем 13725 Дж?

Тетради учащихся собираются для последующей проверки учителем, и результаты анализируются на следующем уроке.

4. Итог урока:

Учитель: «Дорогие ребята! Наш семинар подходит к концу, мы благодарим всех выступавших перед нами. А я еще раз хочу обратить ваше внимание на тему нашего семинара «Решение задач с физическим содержанием. » Таким задачам много внимания уделяется в экзаменационных заданиях и решение этих задач вызывает ряд затруднений, поэтому мы, сегодня уделили внимание именно заданиям такого вида».

а) Проанализировать вместе с учащимися работу групп, указать ошибки, недочёты, отметить положительные моменты.

б) Повторить физические и математические формулы, используемые в предложенных задачах.

в) Выставить отметки за работу на уроке.

5. ДОМАШНЕЕ ЗАДАНИЕ.

Учащимся даётся задание найти в интернете в ОТКРЫТОМ БАНКЕ ЗАДАНИЙ другие виды задач с физическим содержанием и на последующих уроках рассмотреть их решение.

Список

используемой литературы.

  • А.Г.Мордкович «Алгебра и начала анализа» (учебник и задачник), «Мнемозина», 2006г.

  • Г.Я.Мякишев «Физика 11 класс», М. «Просвещение»,2005г.

  • А.П.Власова «Тесты по математике. ЕГЭ», АСТ,2011г.

  • К.Р.Лысенко «Сборник тестов ЕГЭ», 2010г.

МЦКО

О чем говорит сумма баллов, набранных выпускником на профильном ЕГЭ по математике? Какие задания самые сложные? Что нужно знать, чтобы правильно их выполнить? Подробнее об этом рассказывают эксперты Московского центра качества образования: заместитель председателя предметной комиссии ЕГЭ по математике города Москвы Марина Черняева и эксперт предметной комиссии Мария Шабанова.

Сумма баллов как показатель уровня подготовки выпускников

Свидетельство, выдаваемое после ЕГЭ, содержит лишь информацию о сумме баллов, которую выпускник набрал по результатам того или иного экзамена. Эти цифры свидетельствуют об уровне подготовки школьника. Специалисты условно выделяют пять групп. Пороговые значения баллов каждый год немного меняются, но перечень и характеристика групп остаются прежними. Приведем пороговые баллы экзамена 2020 года:

  • 0-6 первичных или 0-27 тестовых баллов: школьники с недостаточным уровнем подготовки для получения аттестата о среднем общем образовании;
  • 7-10 первичных или 33-50 тестовых баллов: освоившие школьный курс математики на базовом уровне, но не обладающие достаточно устойчивыми навыками для успешного продолжения математического образования по техническим специальностям;
  • 11-13 первичных или 56–68 тестовых баллов: имеющие базовый уровень подготовки для освоения базового курса высшей математики в вузе;
  • 14-22 первичных или 70–86 тестовых баллов: имеющие повышенный уровень подготовки, достаточный для продолжения образования по направлениям и специальностям, связанным с освоением специальных математических дисциплин для решения профессиональных задач;
  • 23-32 первичных или 88–100 тестовых баллов: имеющие высокий уровень подготовки, достаточный для дальнейшей специализации в области математики на профессиональном уровне.

Если результаты пробных испытаний пока не соответствуют баллам интересующей выпускника группы, не стоит расстраиваться. Во-первых, еще есть время подготовиться, причем к решению даже самых сложных задач экзамена. А во-вторых, нужно помнить, что это лишь примерная рамка. Она будет скорректирована по результатам экзамена 2021 года. Кроме того, она носит рекомендательный характер. Для уточнения информации советуем заглянуть на сайт приемной комиссии вуза, в который ребенок планирует поступать.

О самых сложных заданиях

В контрольные измерительные материалы входят задания разных уровней сложности: базового (№1-8), повышенного (№9-17) и высокого (№18-19). Самые трудные задачи экзамена – №18 и №19. Их выполнение позволяет набрать до четырех первичных баллов.

Традиционно задание №18 включает алгебраическую задачу на нахождение значений параметров. Чтобы с ним справиться, необходимы не только навыки решения уравнений, неравенств и их систем, аналитического исследования свойств функций, но и гибкость мышления, готовность привлекать методы и средства из разных разделов математики. Одна из проблем решения данного задания – проблема выбора способа, который быстрее всего приведет к правильному результату. Убедиться в этом можно, если посмотреть разбор решения задания №18 ЕГЭ-2020 (резервный день).

Задание №19 представляет собой задачу на числа, для решения которой вполне достаточно знаний математики в объеме общеобразовательной программы. Главная ее сложность состоит в необходимости обнаружить скрытые закономерности на основе экспериментов с числами, придумать идею решения, а затем построить и исследовать подходящую модель в отношении описанной в задаче ситуации. Разбор задачи № 19 ЕГЭ-2020 представляет доцент МФТИ Борис Трушин.

Задание №18

Практика показывает, что зачетные баллы за задание №18 получают участники экзамена не только с повышенным и высоким уровнем математической подготовки, но и с базовым. Даже если выпускник не готовился к решению задачи с параметрами специально, советуем не отказываться от нее. Некоторые задания такого типа состоят из материалов основной школы и ненамного сложнее задач ОГЭ (посмотреть пример задачи ОГЭ с параметром). Так что и здесь получить 1–2 балла вполне реально.

Советы по подготовке к решению задания №18

Приступать к освоению методов и приемов решения данного задания лучше после выполнения задач №13 и №15.

Для начала рекомендуем попробовать решить простое уравнение или неравенство с параметром из школьного учебника (линейное, квадратное, дробно-рациональное, тригонометрическое и т. п.) и сравнить с решением аналогичного уравнения или неравенства без параметра. Стоит следить за тем, как неопределенность значения коэффициента влияет на ход решения.

После того как выпускник освоится с решением простейших задач с параметром, можно приступать к решению задач из открытых банков заданий. Для начала лучше выбирать задачи, допускающие решение аналитическим методом. Подборку таких упражнений (аналитическое решение уравнений, неравенств и их систем) можно посмотреть здесь.

Особенность задач с параметрами заключается в возможности привлечь к их решению различные идеи и методы: построение изображения множества решений в координатах или использование знаний видов и свойств элементарных функций.

Нужно стараться расширять арсенал идей и применяемых методов. Для этого можно использовать специальные пособия (например, пособие С. А. Шестакова), видеоуроки, открытые банки заданий с разбором решений. Чем богаче арсенал, тем выше вероятность получить наивысший балл за это задание.

Задание №19

Средний процент выполнения задания №19 гораздо выше, чем заданий №14, 16 и 18.

В чем же секрет? Дело в том, что задание №19 состоит из трех задач, и лишь третья – высокого уровня сложности, первые же две – базового. Они доступны не только всем участникам экзамена, но и широкому кругу любителей поломать голову над математической задачей.

Попробуйте сами:

На доске написано n единиц, между некоторыми из них поставили знаки + и посчитали сумму. Например, если изначально было написано n = 12 единиц, то могла получиться такая сумма:

1 + 11 + 11 + 111 + 11 + 1 + 1 = 147.

а) Могла ли сумма равняться 150, если n = 60?

б) Могла ли сумма равняться 150, если n = 80?

А теперь проверьте правильность своих выводов здесь.

Советы по подготовке к решению задания №19

Части А и Б задания №19 требуют ответа на вопрос «Может ли?». Положительный ответ должен быть подтвержден примером, а отрицательный – демонстрацией противоречия.

«Артподготовкой» к конструированию примеров может служить решение заданий №19 и 20 базового ЕГЭ по математике: «Числа и их свойства». Это позволит повторить свойства и способы записи целых чисел, а также создать арсенал приемов конструирования примеров.

Вооружившись этими знаниями и опытом, можно осилить решение задачи №19 профильного ЕГЭ в той части, где требуется дать положительный ответ на вопрос «Может ли?». Не стоит сразу же смотреть ответ, он ничего не расскажет о логике рассуждений, предшествующей появлению примера. Лучше прийти к нему самостоятельно.

Долгие и безуспешные попытки сконструировать подходящий пример наводят на мысль об отрицательном ответе на поставленный вопрос. Здесь стоит вспомнить и применить метод «от противного», который школьники изучали в курсе геометрии. Нужно допустить, что пример существует и пытаться вести рассуждение так, чтобы всем стало ясно: подобное допущение приводит к противоречию. Если опыта применения данного метода мало, лучше начать с прочтения разбора готовых решений, а затем перейти к аналогичным задачам и попытаться решить их самостоятельно.

Ресурсы для самоподготовки

Вариантов для самоподготовки к экзаменам сегодня достаточно. Так, например, на ресурсах Московского центра качества образования можно посмотреть консультации, которые проводят ведущие эксперты предметных комиссий ЕГЭ города Москвы.

Кроме того, Московский центр качества образования совместно с Московским образовательным телеканалом запустили новый еженедельный проект – «Субботы московского выпускника» для учащихся 9-х и 11-х классов. Ведущие эксперты МЦКО в прямом эфире Mosobr.tv разбирают решения заданий ЕГЭ и ОГЭ, а также проводят видеоконсультации по различным темам для подготовки к государственной итоговой аттестации. Школьники могут сами выбрать тему ближайшей программы. Для этого необходимо принять участие в голосовании, которое проходит каждую неделю с понедельника по среду.

Для подготовки к ЕГЭ также можно использовать демоверсии, онлайн-уроки, дистанционные консультации и вебинары с разбором заданий от экспертов МЦКО – они доступны в записи на сайте Московского центра качества образования.

А с помощью самодиагностики МЭШ можно проверить свой уровень подготовки по каждому школьному предмету. На сайте доступны работы четырех уровней сложности: стартового, базового, профильного и олимпиадного.

Главное – не бояться трудностей, грамотно выстраивать процесс подготовки, не упускать драгоценное время, и тогда отличный результат ЕГЭ перестанет быть призрачной мечтой.

Источник: Activity.edu

Вопросы по алгебре с решениями и ответами для 11-го класса

    вопросов

  1. Заполните квадрат квадратичной функции f, заданной формулой

    f (x) = 2x 2 — 6x + 4

  2. Найдите точку (точки) пересечения параболы с уравнением y = x 2 — 5x + 4 и прямой с уравнением y = 2x — 2
  3. Найдите константу k так, чтобы: -x 2 — (k + 7) x — 8 = — (x — 2) (x — 4)
  4. Найдите центр и радиус окружности с помощью уравнения x 2 + y 2 -2x + 4y — 11 = 0
  5. Найдите постоянную k так, чтобы квадратное уравнение 2x 2 + 5x — k = 0 имело два действительных решения.
  6. Найдите постоянную k так, чтобы система двух уравнений: 2x + ky = 2 и 5x — 3y = 7 не имела решений.
  7. Разложите выражение на множители 6x 2 — 13x + 5
  8. Упростим i 231 , где i — мнимая единица, определяемая как: i = √ (-1).
  9. Каков остаток от деления f (x) = (x — 2) 54 на x — 1?
  10. Найдите b и c так, чтобы парабола с уравнением y = 4x 2 — bx — c имела вершину в (2, 4)?
  11. Найдите все нули многочлена P (x) = x 3 — 3x 2 — 10x + 24, зная, что x = 2 является нулем многочлена.
  12. Если x — целое число, какое наибольшее значение x удовлетворяет условию 5 <2x + 2 <9?
  13. Упростить | — x 2 + 4x — 4 |.
  14. Найдите константу k так, чтобы прямая с уравнением y = kx касалась окружности с уравнением (x — 3) 2 + (y — 5) 2 = 4.

Решения вышеуказанных вопросов

  1. f (x) = 2 (x 2 — 3x) + 4: множитель 2 в первых двух членах

    = 2 (x 2 — 3x + (-3/2) 2 — (-3/2) 2 ) + 4: сложить и вычесть (-3/2) 2
    = 2 ( x — 3/2)) 2 — 1/2: полный квадрат и сгруппированные термины

  2. 2x — 2 = x 2 — 5x + 4: заменить y на 2x — 2

    x = 1 и x = 6: решение квадратного уравнения

    (1, 0) и (6, 10): точки пересечения

  3. -x 2 — (k + 7) x — 8 = — (x — 2) (x — 4): задано

    -x 2 — (k + 7) x — 8 = -x 2 + 6x — 8

    — (k + 7) = 6: два многочлена равны, если их соответствующие коэффициенты равны.
    k = -13: решите вышеуказанное для k

  4. x 2 — 2x + y 2 + 4y = 11: сложите члены в x вместе и члены в y вместе

    (x — 1) 2 + (y + 2) 2 — 1 — 4 = 11

    (x — 1) 2 + (y + 2) 2 = 4 2 : запишите уравнение круга в стандартной форме

    center (1, -2) и radius = 4: определить центр и радиус

  5. 2x 2 + 5x — k = 0: задано

    дискриминант = 25-4 (2) (- k) = 25 + 8k

    25 + 8k> 0: квадратные уравнения имеют 2 вещественных решения, когда дискриминант положительный

    к> -25/8

  6. Определитель = -6 — 5 тыс.

    -6 — 5k = 0: когда определитель равен нулю (и уравнения независимы), система не имеет решения

    k = -6/5: решить относительно k

  7. 6x 2 — 13x + 5 = (3x — 5) (2x — 1)

  8. Обратите внимание, что i 4 = 1

    Отметим также, что 231 = 4 * 57 + 3

    Отсюда i 231 = (i 4 ) 57 * i 3
    = 1 57 * -i = -i

  9. остаток = f (1) = (1-2) 54 = 1: теорема об остатке

  10. h = b / 8 = 2: формула для координаты x вершины

    b = 16: решить относительно b

    y = 4 для x = 2: вершина точки является решением уравнения параболы

    4 (2) 2 900 10-16 (2) — с = 4

    c = -20: решить относительно c

  11. разделите P (x) на (x — 2), чтобы получить x 2 — x + 12

    P (x) = (x 2 — x + 12) (x — 2)

    = (x — 4) (x + 3) (x — 2): множить квадратичный член

    нули: 4, -3 и 2

  12. 5 <2x + 2 <9: дано
    3/2 <х <7/2
    наибольшее целое значение равно 3 (целое число меньше 7/2)

  13. A пересечение B = {3}: общий элемент для A и B равен 3

    Объединение B = {2, 3, 6, 8, 10, 5, 7, 9}: все элементы A и B находятся в объединении.Элементы, общие для A и B, указываются только один раз, поскольку это набор.

  14. | — x 2 + 4x — 4 | : данный

    = | — (x 2 + 4x — 4) |

    = | — (x — 2) 2 |

    = (x — 2) 2

  15. (x — 3) 2 + (y — 5) 2 = 4: задано

    (x — 3) 2 + (kx — 5) 2 = 4: заменить y на kx

    x 2 (1 + k 2 ) — x (6 + 10k) + 21 = 0: развернуть и записать квадратное уравнение в стандартной форме.
    (6 + 10k) 2 — 4 (1 + k 2 ) (21) = 0: для того, чтобы окружность и прямая y = kx касались, дискриминант вышеуказанного квадратного уравнения должен быть равен нулю. .

    16k 2 + 120k — 48 = 0: развернуть вышеприведенное уравнение

    k = (-15 + √ (273) / 4, k = (-15 — √ (273) / 4): решите вышеуказанное квадратное уравнение.

Репетитор по математике для 11 класса, Онлайн-справка и практика

Не видите то, что вам нужно?
Не волнуйтесь, попробуйте поискать по всем нашим темам

Искать по всему StudyPug

Сложна ли математика в 11 классе?

Математика в 11 классе может быть сложной задачей, и она определенно проверит ваши математические навыки.Из-за этого может казаться, что математика в 11 классе — это сложно и утомительно. Но не бойтесь — мы, в StudyPug, постарались ответить на все ваши вопросы и проблемы! От таких тем, как квадратичные функции до сигма-нотации, до разгадки формулы вершин (и даже больше!), Мы здесь, чтобы помочь вам с математикой для 11-го класса. Наши тщательно разработанные учебные пособия по математике для 11 класса подходят для всех, поэтому независимо от того, являетесь ли вы учеником, которому требуется переподготовка по основам, или тем, кто хочет опередить игру, у нас есть материалы, охватывающие математику для 11 класса для чайников, а также более продвинутые. , сложное содержание.Независимо от уровня сложности, мы здесь, чтобы помочь вам на каждом этапе пути с помощью пошаговых примеров и множества практических вопросов. Математика в 11 классе не должна быть трудной! С помощью наших уроков по повторению и систематической практики, вы обнаружите, что справляетесь с трудными вопросами с легкостью и уверенностью в кратчайшие сроки!

Как подготовиться к 11 классу по математике?

Чувствуете себя неподготовленным к еще одному году математики? Не знаете, как и с чего начать заниматься математикой в ​​11 классе? В StudyPug есть все инструменты и ресурсы, которые помогут вам подготовиться к 11 классу математики!

Понимание математики для 11 класса потребует времени, и это может быть сложно, если вы встречаетесь со своим репетитором только раз в неделю или чувствуете, что вам нужно освежить свои знания по темам, ранее затронутым в классе.Здесь, в StudyPug, наши репетиторы по математике для 11-х классов и весь наш контент всегда в вашем распоряжении, когда они вам понадобятся. Благодаря круглосуточной поддержке и тысячам уроков, которые у вас под рукой, мы здесь, чтобы помочь вам улучшить свое понимание математических тем 11 класса и подготовиться к предстоящему тесту по математике в 11 классе и / или к экзамену по математике в 11 классе. По-прежнему чувствуете, что вам нужна дополнительная поддержка и справочная информация по некоторым темам, изучаемым математикой в ​​11 классе? Некоторые из тем, которые вы изучаете в 11 классе по математике (например,г. Квадратичные отношения, завершающие квадрат) мы начали вводить в 10 классе по математике. Математика 10-го класса является обязательным курсом для 11-го класса, и мы настоятельно рекомендуем хорошо владеть знаниями и навыками, полученными в 10-м классе, чтобы помочь вам преуспеть в математике в 11-м классе. Перейдите на нашу страницу математики для 10-го класса, чтобы начать исправление прямо сейчас!

Как использовать StudyPug для изучения математики в 11 классе?

Нашим намерением для StudyPug было создание онлайн-компаньона, работающего круглосуточно и без выходных, чтобы удовлетворить все ваши потребности в обучении математике в 11 классе.Мы тщательно разработали наш веб-сайт, чтобы он был интуитивно понятным и легким для навигации, но мы также включили этот раздел для тех из вас, кто посещает или пробует нас впервые. Имейте в виду, что если вы студент, который все еще не уверен, подходит ли вам StudyPug, вы все равно можете попробовать наш продукт / услугу, начав с бесплатных уроков, которые доступны сразу и без проблем.

В качестве общего правила, прежде чем переходить к какой-либо из наших тем по математике в 11 классе, мы рекомендуем начать с оценки ваших предварительных знаний.Это просто делается путем проверки вашего знакомства с концепциями, которые, по нашему мнению, вы должны относительно хорошо владеть. Почти все наши темы начинаются с описания основных концепций, которые вам следует знать, а также других связанных, актуальных концепций.

Если вы точно знаете, что ищете, мы рекомендуем сразу перейти к использованию нашей поисковой системы. Используйте это, чтобы отточить концепции, темы или вопросы, которые вы считаете наиболее трудными. Воспоминание; не бойтесь, если вы чувствуете, что вам нужно начать с некоторых элементарных основ, которые вы изучили ранее в 10 классе по математике или где-либо еще.

После того, как вы определили тему, над которой вам требуется дополнительная работа, наши короткие, но информативные видеоуроки проведут вас через примеры (от простых до сложных), чтобы конкретизировать концепции, которые вам необходимо изучить, а также советы и стратегии изучения. Смотрите и пересматривайте эти уроки столько раз, сколько вам нужно! Мы также предоставляем перефразированную расшифровку сути того, о чем говорилось в видео, если вы предпочитаете изучать и исправлять этот способ.

Последний, но не последний из этих шагов, мы призываем вас проверить свои знания и навыки! Мы подготовили множество вопросов по каждой теме, чтобы вы усвоили их и довели до совершенства! Посмотрите, насколько хорошо вы научились, и получите больше практики из самых разных и разнообразных (т.е. от простого к сложному) вопросов.

Ищете более увлекательное чтение и способ выучить математику в 11 классе? Загляните в наш блог, чтобы узнать о последовательности Фибоначчи и Ким Кардашьян!

Как сдать 11 класс по математике?

Готовитесь к тесту или экзамену по математике в 11 классе? Вы попали в нужное место! На нашей домашней странице https://www.studypug.com/ca/grade11 есть все необходимое для изучения математики в 11 классе. Мы, StudyPug, существуем, чтобы помочь вам в достижении ваших академических целей. Итак, готовитесь ли вы к тесту по главе в единичном круге или вам требуется быстрый обзор нескольких тем для большого заключительного экзамена, мы хотим убедиться, что вы чувствуете себя уверенно при выполнении всех своих оценок.

В StudyPug мы проводим многочисленные уроки математики для 11 классов, охватывающие несколько провинциальных и международных учебных программ. Мы также создаем контент на основе нескольких известных учебников, поэтому мы делаем все возможное, чтобы помочь вам сосредоточиться на многочисленных темах и типах вопросов, с которыми вы, возможно, столкнетесь в математике в 11 классе.

Помните: изучение и подготовка к сдаче математики в 11 классе не произойдет в одночасье! Так что не расстраивайтесь, если одни главы или проблемы кажутся вам более сложными, чем другие.Ключ состоит в том, чтобы сначала хорошо усвоить основные концепции, а затем выработать привычку применять свои знания на практике. Предлагая в общей сложности более 290 уникальных практических вопросов по математике для 11-го класса, мы с готовностью дополняем ваше обучение в классе, чтобы не просто сдать экзамен по математике в 11-м классе, но и получить отличные оценки в процессе. Наш перечень практических вопросов доступен вам сразу же и должен облегчить выполнение заданий по математике для 11-х классов.

11 приложений, которые заставят вас немного меньше ненавидеть математику

Помните учебники по математике из старых добрых школьных времен? Те, которые предлагали пошаговые решения каждой проблемы в вашем учебнике? Пришло время попрощаться с этими руководствами и воспользоваться приложениями, которые позволяют решать сложные математические задачи так же просто, как щелкать по картинке!

Сегодня вы можете выбирать из средств обучения, которые помогут вам отслеживать формулы и изучать алгебру, до приложений, которые позволяют решать уравнения, просто наводя камеру.В различных магазинах приложений также есть ряд недорогих или бесплатных научных калькуляторов, что делает этот удобный инструмент гораздо более доступным, чем в те времена, когда он стоил целое состояние.

Это ни в коем случае не исчерпывающий список, но мы попробовали несколько приложений и пришли к выводу, что они лучше всех делают математику (почти) увлекательной.

1. Photomath

Photomath, вероятно, лучшее приложение для решения математических задач. Он использует дополненную реальность, что означает, что вы можете просто навести камеру на любой лист бумаги с уравнением или арифметической задачей, и он найдет решение.Конечно, есть ограничения. На данный момент приложение не может распознать рукописные проблемы, но хорошо распознает печатные. Он также не может решать квадратные уравнения, функциональные уравнения или математические задачи.

Тем не менее, приложение отлично справляется с основными арифметическими задачами и алгебраическими уравнениями. Приложение показывает решения на экране и показывает подсказку «Шаги», показывающую, как оно решило проблему. Он также ведет журнал всех решенных уравнений, поэтому при необходимости вы можете быстро обратиться к более старой проблеме.

Photomath распространяется бесплатно на iOS и Windows Phone. Ожидается, что приложение для Android появится в следующем году, сообщается на сайте разработчика.

2. Solve4x

Это бесплатное приложение для iOS поставляется с решателем уравнений, в котором вы можете вручную ввести уравнение для его решения или сделать снимок и автоматически обработать все уравнение. Вы также можете использовать фотографию, которая уже была сохранена в галерее. Он работает с печатным текстом, и даже в этом случае текст может быть искажен, поэтому иногда требуется небольшое легкое редактирование текста после того, как снимок сделан.Одним из ограничений является то, что приложение не поддерживает уравнения со скобками. Приложение решает уравнения — идея заключается в том, что родители могут использовать его для проверки результатов, которые получают их дети, без необходимости быть в курсе сложной алгебры, хотя в этом случае вы можете держать своих детей подальше от смартфонов.

Solve4x распространяется бесплатно на iOS.

3. iMat Mathematics

iMat Mathematics позволяет вводить уравнения и решать их за вас. Платная версия может решать более широкий круг уравнений, чем Photomath.Помимо этого, приложение также включает в себя различные учебные модули, которые мы обсудим более подробно в разделе ниже. Единственным недостатком является то, что, в отличие от Photomath, с iMat Mathematics вы должны вводить уравнения вручную — вы не можете просто сфотографировать уравнения.

iMat Mathematics доступен бесплатно для iOS и Android. Вы можете разблокировать профессиональную версию, купив в приложении.

4. MyScript Calculator

Это приложение распознает ваш почерк, поэтому вы можете рисовать уравнения на экране, и оно сразу их решает.Он поддерживает основную арифметику, квадратные и кубические корни, кроме тригонометрии, логарифмов и процентов. Вы также можете нарисовать «2 +? = 10», и он подскажет вам правильный ответ.

Нам очень нравится это приложение, но оно не всегда правильно распознает почерк. Заставить его выполнить простое вычисление корня куба было проблемой, потому что он не мог правильно распознать наш ввод. Но когда это действительно работает, это приложение очень удобно.

MyScript Calculator распространяется бесплатно на iOS и Android.

5. PCalc

В iOS App Store есть несколько отличных приложений для научных калькуляторов, но PCalc берет верх, потому что у него также есть красивый виджет Центра уведомлений. Это означает, что вам даже не нужно запускать приложение, когда вам нужно выполнить вычисления — просто проведите вниз от верхнего края экрана, независимо от того, что вы делаете, и приступайте к работе.

PCalc доступен для рупий. 620 на iOS, но есть и бесплатная версия для barebones на случай, если вы захотите попробовать приложение перед покупкой.

6. Научный калькулятор

Для пользователей Android это, вероятно, лучшая альтернатива. Приложение включает в себя ряд функций, включая тригонометрию, логарифмы, экспоненциальные функции, и включает в себя историю уравнений, чтобы вы могли видеть, какую работу вы проделали для достижения своих результатов. Подсветка синтаксиса уравнения, выделение скобок и отдельные режимы для научных и инженерных расчетов делают его отличным выбором, но, прежде всего, это бесплатное приложение также не содержит рекламы.

Научный калькулятор теперь доступен для Android бесплатно.

7. Научный калькулятор (для Windows Phone)

Хотя это приложение имеет то же имя, что и наше приложение для Android, оба приложения от разных компаний выглядят и работают по-разному. Тем не менее, это приложение имеет приятный интерфейс, функции от экспоненциальных до логарифмических и тригонометрических. В нем также есть отдельная вкладка для истории, где вы можете увидеть выполненные вами расчеты.

Пользователи Windows Phone могут бесплатно загрузить Scientific Calculator.

8. Графический калькулятор

Настоящий графический калькулятор, такой как TI-84, по-прежнему будет стоить вам около 100 долларов, но вы найдете множество приложений с той же функциональностью. Это приложение от Mathlab — одно из самых хороших, которые мы видели, и, похоже, оно тоже очень хорошо работает. Более 10000 человек в Play Store оценили его на 5 звезд — и помимо функциональности, которую вы найдете во многих различных приложениях, нам понравилось это приложение за его дизайн, который лучше, чем у большинства аналогичных приложений.Вы также можете попробовать BisMag Calculator 3D. Это приложение имеет аналогичные функции, но также включает в себя решатель уравнений и конвертер валют и единиц, а также графический калькулятор. Это приложение не будет полезно для всех, но если описанные функции соответствуют вашим потребностям, то это отличный вариант, к тому же бесплатный.

Пользователи Android могут получить Графический калькулятор бесплатно. Это приложение недоступно для iOS, но этот бесплатный графический калькулятор является хорошей альтернативой, хотя выглядит не так хорошо.

iMat Mathematics

Это приложение (также упомянутое выше) хорошо объясняет различные темы, такие как алгебра, геометрия, тригонометрия и исчисление. Приложение дает вам краткое определение концепции, сопровождаемое некоторыми примерами или иллюстрациями для ее объяснения. Если в понятие включены какие-либо сложные термины, в приложении есть ссылка на простое объяснение в конце темы. Например, в теме «Тела вращения» упоминается «Усеченный конус». Ссылка в конце темы открывает статью, объясняющую, что такое усеченный конус.

iMat Mathematics также предоставляет ссылку на Википедию в конце каждой записи и позволяет выполнять вычисления с использованием Wolfram Alpha. Все они открываются в приложении, поэтому нет необходимости переключаться между задействованными приложениями для изучения какой-либо темы. Бесплатное приложение позволяет изучить множество базовых концепций, в то время как расширенные концепции доступны в профессиональной версии приложения за рупий. 190 на iOS, а в качестве рупий. 300 покупок в приложении на Android.

iMat Mathematics доступен бесплатно для iOS и Android.

9. Khan Academy

Видеоуроки Khan Academy заслуженно известны и охватывают различные предметы, включая математику. Все, что вам нужно сделать, это установить приложение, выбрать тему и начать просмотр обучающих видео. Приложение также включает в себя практические вопросы, но они также представлены в виде видео, поэтому вам нужно будет записать их, чтобы решить.

Приложения Khan Academy доступны бесплатно для iOS. Сторонние приложения, которые позволяют просматривать видео Khan Academy, доступны на Android и Windows Phone.

10. Meritnation

Приложение Meritnation является цифровым эквивалентом путеводителя для индийских студентов. Он охватывает учебные программы от 6 до 12 для CBSE, ICSE и государственных советов Махараштры, Кералы и Тамил Наду. Бесплатно для Android и iOS математический раздел включает задачи из каждой главы и дает вам решение. Это будет полезно для студентов из Индии, которые хотят использовать приложение специально для подготовки к экзаменам. Приложение требует обязательной регистрации и запрашивает номер вашего мобильного телефона, а большая часть расширенного контента также разблокируется через веб-сайт Meritnation.Это означает, что вы можете разблокировать доступ на нескольких платформах, но в остальном это немного неудобно.

Meritnation бесплатно для iOS и Android.

11. Mathematicus

Mathematicus — хорошее приложение, если у вас проблемы с запоминанием математических формул. Приложение служит базой данных для всех важных формул, и вы можете просто запустить его в любое время и искать то, что вам нужно. Эти формулы складываются по темам, что означает, что вы можете найти все формулы тригонометрии в одной группе и так далее.Приложение не выполняет слишком много функций, но для конкретного использования поиска формул это лучший выбор.

Mathematicus распространяется бесплатно на Windows Phone.

Какие ваши любимые математические приложения? Сообщите нам об этом в комментариях.

15 самых сложных вопросов по SAT математике

Хотите проверить себя, отвечая на самые сложные вопросы по математике SAT? Хотите знать, что делает эти вопросы такими сложными и как их лучше всего решать? Если вы готовы по-настоящему погрузиться в математический раздел SAT и нацелиться на этот высший балл, то это руководство для вас.

Мы собрали то, что мы считаем , из 15 самых сложных вопросов для текущего SAT , со стратегиями и ответами на каждый из них. Все это сложные вопросы SAT Math из практических тестов SAT College Board, а это значит, что их понимание — один из лучших способов учиться для тех из вас, кто стремится к совершенству.

Изображение: Соня Севилья / Викимедиа

Краткий обзор SAT Math

Третий и четвертый разделы SAT всегда будут математическими разделами .Первый математический подраздел (с меткой «3») не позволяет использовать калькулятор , , а не , а второй математический подраздел (с меткой «4») разрешает использование калькулятора. Однако не беспокойтесь о разделе без калькулятора: если вам не разрешено использовать калькулятор для ответа на вопрос, это означает, что вам не нужен калькулятор, чтобы ответить на него.

Каждый математический подраздел расположен в порядке возрастания сложности (где чем больше времени требуется на решение задачи и чем меньше людей ответят на нее правильно, тем сложнее).В каждом подразделе вопрос 1 будет «легким», а вопрос 15 — «сложным». Однако возрастающая сложность сбрасывается с простого на сложный на сетке.

Следовательно, вопросы с несколькими вариантами ответов упорядочены по возрастающей сложности (вопросы 1 и 2 будут самыми легкими, вопросы 14 и 15 будут самыми сложными), но уровень сложности будет сброшен для секции сетки (то есть вопросы 16 и 17 снова будут будьте «легкими», и вопросы 19 и 20 будут очень сложными).

Таким образом, за очень немногими исключениями, самые сложные математические задачи SAT будут сгруппированы в конце сегментов с несколькими вариантами ответов или во второй половине вопросов сетки. Однако, помимо места в тесте, у этих вопросов есть еще несколько общих черт. Через минуту мы рассмотрим примеры вопросов и способы их решения, а затем проанализируем их, чтобы выяснить, что общего у этих типов вопросов.

Но сначала: следует ли вам прямо сейчас сосредоточиться на самых сложных математических вопросах?

Если вы только начинаете подготовку к учебе (или если вы просто пропустили этот первый важный шаг), обязательно остановитесь и пройдите полный практический тест, чтобы определить свой текущий уровень оценок. Ознакомьтесь с нашим руководством по всем бесплатным практическим тестам SAT, доступным в Интернете, а затем сядьте и сдавайте все сразу.

Абсолютно лучший способ оценить свой текущий уровень — просто пройти практический тест SAT, как если бы он был настоящим, соблюдая строгий график и работая без перерыва только с разрешенными перерывами (мы знаем — вероятно, это не ваш любимый способ провести субботу) . Как только вы получите хорошее представление о своем текущем уровне и процентильном рейтинге, вы можете установить контрольные точки и цели для получения окончательного результата по SAT Math.

Если вы в настоящее время набираете баллы в диапазоне 200–400 или 400–600 по SAT Math, лучше всего сначала ознакомиться с нашим руководством по повышению своего балла по математике. в попытке решить самые сложные математические задачи на тесте.

Если, однако, вы уже набрали больше 600 баллов по математике и хотите проверить свои способности на реальном SAT, то обязательно переходите к остальной части этого руководства. Если вы стремитесь к совершенству (или близкому к нему), вам необходимо знать, как выглядят самые сложные вопросы по математике SAT и как их решать.К счастью, именно этим мы и займемся.

ПРЕДУПРЕЖДЕНИЕ: Поскольку количество официальных практических тестов SAT ограничено, вы можете подождать, чтобы прочитать эту статью, пока не попробуете все или большую часть первых четырех официальных практических тестов (поскольку большинство вопросов, приведенных ниже, были приняты из этих тестов). Если вы беспокоитесь о том, чтобы испортить эти тесты, прекратите читать это руководство сейчас; вернитесь и прочтите, когда вы их закончите.

А теперь перейдем к нашему списку вопросов (уууу)!

Изображение: Niytx / DeviantArt

15 сложнейших вопросов по SAT математике

Теперь, когда вы уверены, что должны попытаться ответить на эти вопросы, давайте приступим прямо к делу! Мы собрали 15 самых сложных вопросов по SAT Math, которые вы можете попробовать ниже, а также пошаговые инструкции, как получить ответ (если вы в тупике).

Нет калькулятора Вопросы по SAT по математике

Вопрос 1

$$ C = 5/9 (F-32) $$

Приведенное выше уравнение показывает, как температура $ F $, измеренная в градусах Фаренгейта, соотносится с температурой $ C $, измеренной в градусах Цельсия. Основываясь на уравнении, какое из следующих утверждений должно быть верным?

  1. Повышение температуры на 1 градус по Фаренгейту эквивалентно повышению температуры на 5/9 градусов Цельсия.
  2. Повышение температуры на 1 градус Цельсия эквивалентно повышению температуры на 1.8 градусов по Фаренгейту.
  3. Повышение температуры на 5 долларов / 9 градусов по Фаренгейту эквивалентно повышению температуры на 1 градус Цельсия.

A) только I
B) только II
C) только III
D) только I и II

ОБЪЯСНЕНИЕ ОТВЕТА: Думайте об уравнении как об уравнении для линии

$$ y = mx + b $$

, где в данном случае

$$ C = {5} / {9} (F − 32) $$

или

$$ C = {5} / {9} F — {5} / {9} (32) $$

Вы можете видеть, что наклон графика составляет $ {5} / {9} $, что означает, что при увеличении на 1 градус по Фаренгейту увеличение составляет $ {5} / {9} $ на 1 градус Цельсия.

$$ C = {5} / {9} (F) $$

$$ C = {5} / {9} (1) = {5} / {9} $$

Следовательно, утверждение I верно. Это эквивалентно тому, что увеличение на 1 градус Цельсия равно увеличению на $ {9} / {5} $ градусов по Фаренгейту.

$$ C = {5} / {9} (F) $$

$$ 1 = {5} / {9} (F) $$

$$ (F) = {9} / {5} $$

Поскольку $ {9} / {5} $ = 1.8, утверждение II верно.

Единственный ответ, в котором и утверждение I, и утверждение II являются истинными, — это D , но если у вас есть время и вы хотите быть абсолютно внимательными, вы также можете проверить, соответствует ли утверждение III (увеличение на $ {5} / { 9} $ градус Фаренгейта равен увеличению температуры на 1 градус Цельсия) верно:

$$ C = {5} / {9} (F) $$

$$ C = {5} / {9} ({5} / {9}) $$

$$ C = {25} / {81} (\ which \ is ≠ 1) $$

Увеличение на 5 долларов / 9 градусов по Фаренгейту приводит к увеличению на {25} / {81} долларов, а не на 1 градус Цельсия, и поэтому утверждение III неверно.2 $ -члены должны быть равны с обеих сторон уравнения, $ −8a = 24 $ или $ a = −3 $.

Другой вариант, который более длительный и утомительный, — это попытаться включить все варианты ответов для a и посмотреть, какой вариант ответа делает обе стороны уравнения равными. 2 $
D) Значение не может быть определено на основе предоставленной информации.12 $$

Окончательный ответ: A.

Вопрос 4

Точки A и B лежат на окружности радиуса 1, а длина дуги $ {AB} ↖⌢ $ равна $ π / 3 $. Какая часть окружности окружности равна длине дуги $ {AB} ↖⌢ $?

ОБЪЯСНЕНИЕ ОТВЕТА: Чтобы выяснить ответ на этот вопрос, вам сначала необходимо знать формулу для определения длины окружности.

Длина окружности $ C $ равна $ C = 2πr $, где $ r $ — радиус окружности.Для данной окружности радиусом 1 длина окружности равна $ C = 2 (π) (1) $ или $ C = 2π $.

Чтобы узнать, какая часть окружности составляет длину $ {AB} ↖⌢ $, разделите длину дуги на длину окружности, что даст $ π / 3 ÷ 2π $. Это деление можно представить как $ π / 3 * {1/2} π = 1/6 $.

Дробь $ 1/6 $ также может быть переписана как $ 0,166 $ или 0,167 $.

Окончательный ответ: 1/6 доллара, 0,166 доллара или 0,167 доллара.

Вопрос 5

$$ {8-i} / {3-2i} $$

Если приведенное выше выражение переписать в форме $ a + bi $, где $ a $ и $ b $ — действительные числа, каково значение $ a $? (Примечание: $ i = √ {-1} $)

ОБЪЯСНЕНИЕ ОТВЕТА: Чтобы переписать $ {8-i} / {3-2i} $ в стандартной форме $ a + bi $, вам нужно умножить числитель и знаменатель $ {8-i} / {3- 2i} $ сопряженным, $ 3 + 2i $.2 = -1 $, эту последнюю дробь можно сократить упрощенно до

$$ {24 + 16i-3i + 2} / {9 — (- 4)} = {26 + 13i} / {13} $$

, который упрощается до 2 + i $. Следовательно, когда $ {8-i} / {3-2i} $ переписывается в стандартной форме a + bi, значение a равно 2.

Окончательный ответ: A.

Вопрос 6

В треугольнике $ ABC $ мера $ ∠B $ равна 90 °, $ BC = 16 $ и $ AC $ = 20. Треугольник $ DEF $ похож на треугольник $ ABC $, где вершины $ D $, $ E $ и $ F $ соответствуют вершинам $ A $, $ B $ и $ C $ соответственно, а также каждой стороне треугольника $. DEF $ составляет $ 1/3 $ длины соответствующей стороны треугольника $ ABC $.2} = √ {400-256} = √ {144} = 12 $$

Поскольку треугольник DEF подобен треугольнику ABC, с вершиной F, соответствующей вершине C, мера $ \ angle ∠ {F} $ равна мере $ \ angle ∠ {C} $. Следовательно, $ sin F = sin C $. Со сторон треугольника ABC,

$$ sinF = {\ Against \ side} / {\ hypotenuse} = {AB} / {AC} = {12} / {20} = {3} / {5} $$

Следовательно, $ sinF = {3} / {5} $.

Окончательный ответ: $ {3} / {5} $ или 0,6.

Вопросы SAT по математике, разрешенные калькулятором

Вопрос 7

В неполной таблице выше суммировано количество учащихся-левшей и учащихся-правшей с разбивкой по полу для учащихся восьмых классов средней школы им. Кейзеля.Учениц-правшей в 5 раз больше, чем учениц-левшей, и учеников-правшей в 9 раз больше, чем учениц-левшей. Если в школе 18 учеников-левшей и 122 учащихся-правшей, что из следующего наиболее близко к вероятности того, что случайно выбранный ученик-правша будет женщиной? (Примечание: предположим, что ни один из восьмиклассников не является одновременно правшой и левшой.)

А) 0.410
B) 0,357
C) 0,333
D) 0,250

ОБЪЯСНЕНИЕ ОТВЕТА: Чтобы решить эту проблему, вы должны создать два уравнения, используя две переменные ($ x $ и $ y $) и предоставленную вам информацию. Пусть $ x $ будет количеством учениц-левшей и пусть $ y $ будет количеством учениц-левшей. Используя информацию, приведенную в задаче, количество учащихся-правшей будет составлять 5 долларов США, а количество учащихся-правшей будет составлять 9 лет.Поскольку общее количество студентов-левшей составляет 18, а общее количество студентов-правшей — 122, система уравнений ниже должна быть верной:

$$ x + y = 18 $$

$$ 5x + 9y = 122 $$

Когда вы решите эту систему уравнений, вы получите $ x = 10 $ и $ y = 8 $. Таким образом, из 122 учащихся-правшей 5 * 10, или 50, — девушки. Следовательно, вероятность того, что случайным образом выбранный студент-правша будет женщиной, составляет {50} / {122} $, что с точностью до тысячных составляет 0,410.

Окончательный ответ — А.

Вопросы 8 и 9

Используйте следующую информацию как для вопроса 7, так и для вопроса 8.

Если покупатели входят в магазин со средней скоростью $ r $ покупателей в минуту и ​​каждый остается в магазине в течение среднего времени T $ минут, среднее количество покупателей в магазине, N $, в любой момент времени составляет задается формулой $ N = rT $. Эта связь известна как закон Литтла.

По оценкам владельца магазина Good Deals Store, в рабочее время в магазин заходит в среднем 3 покупателя в минуту, и каждый из них остается в среднем на 15 минут.Владелец магазина использует закон Литтла, чтобы оценить, что в магазине одновременно находится 45 покупателей.

Вопрос 8

Закон Литтла может применяться к любой части магазина, например к определенному отделу или кассовым линиям. Владелец магазина определяет, что в рабочее время примерно 84 покупателя в час совершают покупку, и каждый из этих покупателей проводит в очереди в кассе в среднем 5 минут. Сколько в среднем покупателей в любое время в рабочее время ожидают в очереди у кассы, чтобы совершить покупку в магазине Good Deals Store?

ОБЪЯСНЕНИЕ ОТВЕТА: Поскольку в вопросе говорится, что закон Литтла может применяться к любой отдельной части магазина (например, только к кассе), тогда среднее количество покупателей, $ N $, в очереди к кассе в любой time равно $ N = rT $, где $ r $ — это количество покупателей, заходящих в кассу в минуту, а $ T $ — это среднее количество минут, которое каждый покупатель проводит в очереди.

Поскольку 84 покупателя в час совершают покупку, 84 покупателя в час входят в кассу. Однако это необходимо преобразовать в количество покупателей в минуту (для использования с $ T = 5 $). Поскольку в часе 60 минут, тариф составляет $ {84 \ shoppers \ per \ hour} / {60 \ minutes} = 1,4 $ покупателя в минуту. Используя данную формулу с $ r = 1,4 $ и $ T = 5 $, получаем

$$ N = rt = (1.4) (5) = 7 $$

Таким образом, среднее количество покупателей, $ N $, в очереди на кассу в любое время в рабочее время равно 7.

Окончательный ответ 7.

Вопрос 9

Владелец магазина Good Deals Store открывает новый магазин в другом конце города. По оценкам владельца нового магазина, в рабочее время в него заходят в среднем 90 покупателей в час, и каждый из них остается в среднем на 12 минут. Среднее количество покупателей в новом магазине в любой момент времени на какой процент меньше среднего количества покупателей в исходном магазине в любое время? (Примечание: игнорируйте символ процента при вводе ответа.Например, если ответ 42,1%, введите 42,1)

ОБЪЯСНЕНИЕ ОТВЕТА: Согласно исходной информации, оценочное среднее количество покупателей в исходном магазине в любой момент времени (N) составляет 45. В вопросе говорится, что в новом магазине менеджер оценивает, что в среднем 90 покупателей в час (60 минут) заходят в магазин, что эквивалентно 1,5 покупателям в минуту (r). Менеджер также подсчитал, что каждый покупатель остается в магазине в среднем 12 минут (T).Таким образом, по закону Литтла в каждый момент времени в новом магазине в среднем находится $ N = rT = (1.5) (12) = 18 $ покупателей. Это

$$ {45-18} / {45} * 100 = 60 $$

На

процентов меньше, чем среднее количество покупателей в исходном магазине в любое время.

Окончательный ответ — 60.

Вопрос 10

На плоскости $ xy $ точка $ (p, r) $ лежит на прямой с уравнением $ y = x + b $, где $ b $ — константа. Точка с координатами $ (2p, 5r) $ лежит на прямой с уравнением $ y = 2x + b $.Если $ p ≠ 0 $, каково значение $ r / p $?

A) 2/5 долл. США

долл. США

B) 3/4 $

C) 4/3 $

D) $ 5/2 $

ОБЪЯСНЕНИЕ ОТВЕТА: Поскольку точка $ (p, r) $ лежит на прямой с уравнением $ y = x + b $, точка должна удовлетворять уравнению. Подстановка $ p $ вместо $ x $ и $ r $ вместо $ y $ в уравнение $ y = x + b $ дает $ r = p + b $, или $ \ bi b $ = $ \ bi r- \ bi p $.

Аналогично, поскольку точка $ (2p, 5r) $ лежит на прямой с уравнением $ y = 2x + b $, точка должна удовлетворять уравнению.Замена $ 2p $ на $ x $ и $ 5r $ на $ y $ в уравнении $ y = 2x + b $ дает:

$ 5r = 2 (2p) + b $

$ 5r = 4p + b $

$ \ bi b $ = $ \ bo 5 \ bi r- \ bo 4 \ bi p $.

Затем мы можем установить два уравнения, равных $ b $, равным друг другу и упростить:

$ б = р-п = 5р-4п

$

$ 3p = 4r $

Наконец, чтобы найти $ r / p $, нам нужно разделить обе части уравнения на $ p $ и на $ 4 $:

$ 3p = 4r $

$ 3 = {4r} /

долларов США на человека

$ 3/4 = р / п

$

Правильный ответ: B , 3/4 доллара.

Если вы выбрали варианты A и D, возможно, вы неправильно сформировали свой ответ из коэффициентов в пункте $ (2p, 5r) $. Если вы выбрали вариант C, возможно, вы перепутали $ r $ и $ p $.

Обратите внимание, что пока это находится в разделе калькулятора теста SAT, вам совершенно не нужен калькулятор для его решения!

Вопрос 11

Зерновой бункер состоит из двух правых круглых конусов и правого круглого цилиндра с внутренними размерами, представленными на рисунке выше. 2h $$

можно использовать для определения общего объема силоса.2) (5) = ({4} / {3}) (250) π $$

, что примерно равно 1047,2 кубических футов.

Окончательный ответ — Д.

Вопрос 12

Если $ x $ — среднее (среднее арифметическое) для $ m $ и $ 9 $, $ y $ — это среднее значение для $ 2m $ и $ 15 $, а $ z $ — это среднее значение для $ 3m $ и $ 18 $, то что есть среднее значение $ x $, $ y $ и $ z $ в пересчете на $ m $?

A) $ m + 6 $
B) $ m + 7 $
C) $ 2m + 14 $
D) $ 3m + 21 $

ОБЪЯСНЕНИЕ ОТВЕТА: Поскольку среднее (среднее арифметическое) двух чисел равно сумме двух чисел, разделенных на 2, уравнения $ x = {m + 9} / {2} $, $ y = {2m +15} / {2} $, $ z = {3m + 18} / {2} $ верны.2-x- {11} / {4} $$

и

$$ y = k $$

Реальное решение системы двух уравнений соответствует точке пересечения графиков этих двух уравнений на плоскости $ xy $.

График $ y = k $ — это горизонтальная линия, которая содержит точку $ (0, k) $ и трижды пересекает график кубического уравнения (поскольку оно имеет три действительных решения). Учитывая график, единственная горизонтальная линия, которая трижды пересекала бы кубическое уравнение, — это линия с уравнением $ y = −3 $ или $ f (x) = −3 $.2 $$

Динамическое давление $ q $, создаваемое жидкостью, движущейся со скоростью $ v $, можно найти с помощью приведенной выше формулы, где $ n $ — постоянная плотность жидкости. Инженер-авиастроитель использует формулу для определения динамического давления жидкости, движущейся со скоростью $ v $, и той же жидкости, движущейся со скоростью 1,5 $ v $. Каково отношение динамического давления более быстрой жидкости к динамическому давлению более медленной жидкости?

ОБЪЯСНЕНИЕ ОТВЕТА: Чтобы решить эту проблему, вам необходимо задать уравнения с переменными.2 = (2.25) q_1 $$

Следовательно, коэффициент динамического давления более быстрой жидкости равен

$$ {q2} / {q1} = {2.25 q_1} / {q_1} = 2.25 $$

Окончательный ответ — 2,25 или 9/4.

Вопрос 15

Для полинома $ p (x) $ значение $ p (3) $ равно $ -2 $. Что из следующего должно быть истинным относительно $ p (x) $?

A) $ x-5 $ является множителем $ p (x) $.
B) $ x-2 $ — множитель $ p (x) $.
C) $ x + 2 $ является множителем $ p (x) $.
D) Остаток от деления $ p (x) $ на $ x-3 $ равен -2 $.1 $ и не выше), остаток — действительное число.

Следовательно, $ p (x) $ можно переписать как $ p (x) = (x + k) q (x) + r $, где $ r $ — действительное число.

В вопросе указано, что $ p (3) = -2 $, поэтому должно быть верно, что

$$ — 2 = p (3) = (3 + k) q (3) + r $$

Теперь мы можем включить все возможные ответы. Если ответ A, B или C, $ r $ будет $ 0 $, а если ответ D, $ r $ будет $ -2 $.

A. $ -2 = p (3) = (3 + (-5)) q (3) + 0 $
$ -2 = (3-5) q (3) 9000 $ 8 $ -2 = (- 2 ) q (3) $ 90 250

Это могло быть правдой, но только если $ q (3) = 1 $

Б.$ -2 = p (3) = (3 + (-2)) q (3) + 0 $
$ -2 = (3-2) q (3) $
$ -2 = (-1) q ( 3) 90 250 долларов США

Это могло быть правдой, но только если $ q (3) = 2 $

C. $ -2 = p (3) = (3 + 2) q (3) + 0 $
$ -2 = (5) q (3)

$

Это могло быть правдой, но только если $ q (3) = {- 2} / {5} $

D. $ -2 = p (3) = (3 + (-3)) q (3) + (-2) $ 9000 8 $ -2 = (3 — 3) q (3) + (-2) $
-2 = (0) q (3) + (-2) 90 250 долл. США

Это будет всегда верно независимо от того, что такое $ q (3) $.

Из вариантов ответа единственное, что должно быть верным относительно $ p (x) $, — это D, что остаток от деления $ p (x) $ на $ x-3 $ равен -2.

Окончательный ответ — Д.

Хотите улучшить свой результат SAT на 160 баллов? Мы написали руководство о 5 лучших стратегиях, которые вы должны использовать, чтобы улучшить свой результат. Скачать бесплатно сейчас:

Вы заслуживаете того, чтобы вздремнуть, задав эти вопросы.

Что общего у самых сложных вопросов по SAT Math?

Важно понимать, что делает эти сложные вопросы «сложными». Поступая таким образом, вы сможете понять и решить похожие вопросы, когда вы увидите их в день тестирования, а также получите лучшую стратегию для выявления и исправления ваших предыдущих математических ошибок SAT.

В этом разделе мы рассмотрим, что общего у этих вопросов, и дадим примеры каждого типа.Некоторые из причин, по которым самые сложные вопросы по математике являются самыми сложными вопросами по математике, заключаются в том, что они:

# 1: Проверить сразу несколько математических понятий

Здесь мы должны иметь дело с мнимыми числами и дробями одновременно.

Секрет успеха: Подумайте, какую применимую математику вы могли бы использовать для решения задачи, выполняйте по одному шагу за раз и пробуйте каждый метод, пока не найдете тот, который работает!

# 2: задействовать множество шагов

Помните: чем больше шагов вам нужно предпринять, тем легче где-то напортачить!

Мы должны решить эту проблему поэтапно (делая несколько средних значений), чтобы разблокировать остальные ответы в эффекте домино.Это может сбивать с толку, особенно если вы в стрессе или у вас не хватает времени.

Секрет успеха: Не торопитесь, делайте шаг за шагом и перепроверяйте свою работу, чтобы не ошибиться!

# 3: Проверьте концепции, с которыми вы мало знакомы

Например, многие учащиеся менее знакомы с функциями, чем с дробями и процентами, поэтому большинство функциональных вопросов считаются задачами «высокой сложности».

Если вы не разбираетесь в функциях, это может быть сложной проблемой.

Секрет успеха: Просмотрите математические концепции, с которыми вы не так хорошо знакомы, например, функции. Мы предлагаем использовать наши отличные бесплатные руководства по тестированию SAT Math.

# 4: написаны необычно или запутанно

Может быть трудно точно определить, какие вопросы задает , не говоря уже о том, как их решить. Это особенно верно, когда вопрос находится в конце раздела, а у вас не хватает времени.

Поскольку в этом вопросе содержится так много информации без диаграммы, может быть сложно разобраться в ограниченном отведенном времени.

Секрет успеха: Не торопитесь, проанализируйте, что от вас просят, и нарисуйте диаграмму, если это вам поможет.

# 5: Используйте много разных переменных

При таком большом количестве различных переменных очень легко запутаться.

Секрет успеха: Не торопитесь, проанализируйте, что от вас просят, и подумайте, является ли включение цифр хорошей стратегией для решения проблемы (это не относится к вопросу выше, но может быть ко многим другим. SAT переменные вопросы).

На вынос

SAT — это марафон, и чем лучше вы к нему подготовитесь, тем лучше вы будете себя чувствовать в день теста. Знание того, как отвечать на самые сложные вопросы, которые может бросить вам тест, сделает сдачу настоящего SAT намного менее сложной задачей.

Если вам казалось, что эти вопросы были легкими, не стоит недооценивать влияние адреналина и усталости на вашу способность решать проблемы. Продолжая учиться, всегда придерживайтесь надлежащих рекомендаций по времени и старайтесь проходить полные тесты, когда это возможно. Это лучший способ воссоздать реальную среду тестирования, чтобы вы могли подготовиться к реальной сделке.

Если вы считаете, что эти вопросы были сложными, обязательно укрепите свои математические знания, ознакомившись с нашими индивидуальными руководствами по математическим темам для SAT. Здесь вы увидите более подробные объяснения рассматриваемых тем, а также более подробную разбивку ответов.

Что дальше?

Чувствовали, что эти вопросы оказались сложнее, чем вы ожидали? Взгляните на все темы, затронутые в разделе SAT по математике, а затем отметьте, какие разделы были для вас особенно трудными.Затем взгляните на наши индивидуальные руководства по математике, которые помогут вам укрепить любую из этих слабых сторон.

Не хватает времени на сдачу экзамена по математике? Наш гид поможет вам выиграть время и увеличить свой счет.

Хотите набрать наивысший балл? Ознакомьтесь с нашим руководством о том, как набрать 800 баллов по математическому разделу SAT, написанному отличным игроком.

Хотите улучшить свой результат SAT на 160 баллов?

Посетите наши лучшие в своем классе онлайн-классы подготовки к SAT.Мы гарантируем возврат ваших денег , если вы не улучшите свой SAT на 160 или более баллов.

Наши классы полностью онлайн, и их ведут эксперты SAT. Если вам понравилась эта статья, вам понравятся наши классы. Наряду с занятиями под руководством экспертов вы получите индивидуальное домашнее задание с тысячами практических задач, организованных по индивидуальным навыкам, чтобы вы учились наиболее эффективно. Мы также дадим вам пошаговую индивидуальную программу, которой вы будете следовать, чтобы вы никогда не запутались, что изучать дальше.

Попробуйте без риска сегодня:

10 лучших стратегий для улучшения успеваемости по математике

Многие учащиеся и родители просят дать им указатели и методы, чтобы лучше усвоить математику. Вот мой список из 10 лучших, применимый к любому уровню математики.

1) Если вы чего-то не понимаете, сосредоточьтесь на усвоении этой темы, прежде чем переходить к следующей теме. Звучит просто, но абсолютно необходимо. Допустим, студент изучает, например, алгебру.Кроме того, допустим, ему или ей трудно понять, как складывать и вычитать отрицательные и положительные числа. Все мы вначале боремся с этим, так как это проблема для большинства студентов. Некоторые учащиеся в этой ситуации, разочарованные тем, что они «не могут» изучить эту тему, переходят к следующему уроку в надежде, что они смогут понять этот.

Это рецепт катастрофы.

Математика очень похожа на обучение чтению. Если вы не знаете, как звучит ваша буква, у вас нет никакой надежды на то, что вы сможете произносить слова, конечно, вы не сможете прочитать книгу.Все математические курсы преподаются в определенной последовательности, потому что каждая тема основывается на предыдущей. Если у вас возникла проблема с темой, продолжайте работать с ней, пока вы не поймете ее и не сможете успешно решать проблемы. Посмотрите раздел DVD еще раз, посетите занятия, прочитайте книгу и примеры во второй раз или даже возьмите совершенно другую книгу, чтобы объяснить ее по-другому … но что бы вы ни делали, не переворачивайте страницу и не переходите к следующей теме. Если вы это сделаете, вы еще больше расстроитесь и, по всей вероятности, начнете терять надежду.

2) Работайте с примерами задач и проверяйте свои ответы на каждом уроке, чтобы практиковаться. Основная идея серии DVD — «учить на примере», и это самый простой способ выучить математику. После просмотра раздела на DVD и прочтения раздела в учебнике начинайте рабочие примеры с конца главы. Обязательно проработайте задачи, ответы на которые есть в конце книги, и проверьте каждую. Всегда начинайте с самой простой проблемы в своей книге, даже если вы думаете, что ее будет слишком «легко» решить.Очень важно укрепить вашу уверенность в себе. Вот почему уроки DVD начинаются с более простых задач, которые никто не сможет понять. Постепенно работайте над задачами из книги и проверяйте свой ответ на каждую из них. Проработав дюжину или больше задач из раздела (лучше две дюжины), вы готовы переходить к следующему разделу. Многие студенты хотят пропустить урок, чтобы дойти до следующего. Вы не можете просто прочитать раздел в книге по математике и стать экспертом в этом разделе.Вы должны работать над проблемами. Если вы не можете работать с проблемами, значит, вы не готовы двигаться дальше. Хорошая новость заключается в том, что рабочие задачи укрепят вашу уверенность, а уверенность — это 100% название игры в математике.

3) Начиная работать с математической задачей, не «намечайте путь от проблемы к ответу» в уме, прежде чем что-либо записывать. Я вижу это почти каждый день. Очень часто, когда кто-то смотрит на математическую задачу, он пытается «разобраться» в своей голове, прежде чем что-либо записать.Возьмем, к примеру, алгебру. Когда начинающий ученик смотрит на уравнение, он или она испытывает искушение решить уравнение в уме и ничего не записывать. Чаще всего студенты испытывают искушение делать это с помощью задач со словами. Поскольку словесная проблема записана в форме предложения, принято думать, что вы можете «продумать свой путь к ответу». Я скажу вам, что я никогда, никогда не решаю никаких математических задач, не записав их. Всегда.

Что вам нужно сделать, это сначала записать проблему.Затем вы начинаете решать ее, шаг за шагом. Записывайте даже простые вещи. Вам нужно убедиться, что каждый шаг, который вы записываете, совершенно законен. Другими словами, если вы, например, решаете уравнение и вычитаете «10» с обеих сторон … запишите это. Затем на СЛЕДУЮЩЕМ шаге фактически произведите это вычитание. Затем, если вам нужно разделить обе стороны на «2», запишите ЭТО … тогда на СЛЕДУЮЩЕМ шаге фактически сделайте деление. Это дает вам бумажный след для проверки вашей работы, а также позволяет разбить проблему на куски размером с укус.Если вы можете быть уверены, что каждый маленький шаг законен, тогда вы будете в хорошей форме. Если вы попытаетесь сделать слишком много дел одновременно, что является обычным явлением, вы, вероятно, попытаетесь сделать что-то незаконное и попадете в неприятности.

4) Когда вы учитесь и делаете домашнее задание, постарайтесь найти для этого тихое место. Я был самым страшным преступником в этом, пока учился в школе. Раньше я все время слушал музыку, пытаясь делать уроки. Я также слушал телевизор как «фоновый шум» во время учебы.Со временем я понял, что если бы у меня было тихое место без фонового шума, я мог бы гораздо лучше сфокусироваться. Я обнаружил, что при чтении, например … мне пришлось бы прочесть что-то, возможно, 3 или 4 раза, если я слушал что-то другое, но только один раз, если бы я немного затих. Люди любят слушать музыку во время учебы, но я убежден, что если вы этого не сделаете, это будет намного эффективнее. Попробуйте найти тихое место в своем доме или в библиотеке, чтобы делать уроки, и вы сделаете свою работу намного быстрее, потому что вы сможете сосредоточиться и усвоить больше.

5) Если кто-то просит вас о помощи, постарайтесь как можно лучше объяснить ему тему. Это может показаться немного странным для этого списка … но есть одна универсальная правда. Те, кто может учить других, действительно понимают материал. Часто при обучении в группах один член группы отстает и не «понимает». Постарайтесь помочь этому человеку, даже если ваша собственная работа займет больше времени. Вы не только почувствуете, что помогаете кому-то добиться успеха, но и процесс перефразирования информации обратно кому-то и разбиения ее на небольшие куски улучшит ваше собственное понимание.Это поможет вам понять на фундаментальном уровне, что представляют собой камни преткновения для данной темы, что поможет вам в дальнейшем изучении математики.

6) Никогда и никогда не решайте математические задачи ручкой. Это довольно просто. Вы ошибетесь; это только вопрос времени. Когда вы это сделаете, вы захотите полностью стереть свою ошибку и переписать ее. Вам никогда не захочется что-то выцарапать и написать рядом с начертанием. Это приведет к тому, что статью будет трудно читать, а вычеркивания действительно увеличат ваше беспокойство по поводу решения этих проблем.Вам нужна чистая аккуратная бумага с чистым, хорошо продуманным решением.

7) Попробуйте использовать механический карандаш с отдельным ластиком, если можете. У механических карандашей четкие линии, а отдельный ластик позволяет стирать более аккуратно. Нет ничего хуже, чем сделать ошибку и попытаться что-то стереть, а затем размазать это по всей странице. Дешевые ластики сделают это и усложнят вам жизнь. Купите хороший механический карандаш и хороший ластик.

8) Держите свои решения аккуратными и построчно. Всегда решайте задачи вертикально, по одному шагу на каждой строке. Никогда не работайте горизонтально. Для этого может потребоваться больше бумаги, но вам будет гораздо проще следовать своим инструкциям. Что еще более важно, учитель сможет лучше следить за вашей работой, что позволит ему / ей поставить вам частичную оценку. Если есть всего 2 шага, а должно быть 10, вы не получите ни одного балла за свой мыслительный процесс. Записанные шаги рассказывают учителю, о чем вы думаете и как решаете проблему.

9) Не работайте поздно ночью. Я знаю, что все студенты колледжа будут смеяться над этим, но это правда. Я много-много раз пробовал заниматься математикой или физикой поздно ночью, после 12 или 1 часа ночи, но вы просто оказываете себе медвежью услугу. Я часами смотрел на проблемы, потому что я просто не мог заснуть, пока не узнал, как их решить … потом я наконец заснул от сильной усталости … но когда я проснулся, мне стало так просто, как действовать дальше. проблема.Кроме того, я работал над проблемами ночью и получил неправильный ответ, и я знал, что, должно быть, у меня есть глупая ошибка в решении. Обычно я старался найти ее, но часто, когда вы устали, вы просто не можете найти глупую ошибку. На следующее утро, примерно через 5 минут, я мог заметить простую ошибку знака или даже простую ошибку умножения, которая вызвала проблему.

10) Если проблема сама собой поддается, нарисуйте ее картину. Это наиболее применимо для студентов, изучающих тригонометрию, математику и физику, но также применимо к любой проблеме со словами по основам математики или алгебры.Пожалуйста, сделайте себе одолжение и нарисуйте картину того, что описывает проблема, даже если ваша картина проста. Мы — визуальные существа … процесс рисования ситуации заставляет нас усвоить, чего на самом деле требует проблема. Это помогает понять, как действовать дальше. Если вы изучаете физику, вы должны рисовать картинку для каждой решаемой задачи. Если вы работаете в Calculus, обязательно нарисуйте картинки для всех связанных задач с тарифами. Если вы изучаете Исчисление 2 или Исчисление 3, обязательно нарисуйте картину всех ваших трехмерных задач (трехмерные интегралы).Если вы изучаете основы математики и Дженни дает Бобу 2 карандаша, а Боб дает 1 карандаш, нарисуйте эту ситуацию. Это действительно поможет вам понять, как действовать дальше.

Помните, в изучении математики нет серебряной пули. Для этого нужно делать все шаг за шагом и практиковаться. Приведенные выше советы помогут вам в изучении математики и придадут уверенности. А уверенность — это 100% игра в изучении математики любого уровня.


Джейсон Гибсон — основатель MathTutorDVD.com. Вы можете просмотреть его обширную биографию и образование здесь.

20 сложных, но забавных вопросов по математике для начальной школы

Если вы не выросли инженером, банкиром или бухгалтером, велика вероятность, что математика в начальной и средней школе была проклятием вашего существования. Вы будете без устали готовиться неделями к этим глупым стандартизированным тестам — и, тем не менее, придя к экзамену, вы так или иначе не поймете, о чем просили какие-либо уравнения или сложные математические задачи.Поверьте, мы это понимаем.

Хотя логика может привести вас к мысли, что ваши математические навыки естественным образом улучшились с возрастом, прискорбная реальность такова, что, если вы не решаете задачи алгебры и геометрии на ежедневной основе, скорее всего обратное. .

Не верите нам? Затем проверьте свою мудрость с помощью этих сложных математических вопросов, взятых прямо из школьных тестов и домашних заданий, и убедитесь в этом сами.

1.Вопрос: Какое количество парковочных мест занято автомобилем?

Эта сложная математическая задача стала вирусной несколько лет назад после того, как появилась на вступительном экзамене в Гонконге… для шестилетних детей. Якобы у студентов было всего 20 секунд, чтобы решить задачу!

Ответ: 87.

Хотите верьте, хотите нет, но этот «математический» вопрос на самом деле не требует никаких математических вычислений. Если вы перевернете изображение вверх ногами, вы увидите, что вы имеете дело с простой числовой последовательностью.

2. Вопрос: Замените вопросительный знак в указанной выше проблеме на соответствующий номер.

Эту проблему не должно быть слишком трудно решить, если вы много играете в судоку.

Ответ: 6.

Сумма всех чисел в каждой строке и столбце составляет 15! (Кроме того, 6 — единственное число, не представленное из чисел от 1 до 9.)

3. Вопрос: Найдите эквивалентное число.

Эта проблема возникла в результате стандартного теста, проведенного в Нью-Йорке в 2014 году.

Ответ: 9.

Shutterstock

Простите, если вы точно не помните, как работают экспоненты. Чтобы решить эту проблему, вам просто нужно вычесть экспоненты (4-2) и решить для 3 2 , которое расширяется до 3 x 3 и равно 9.

4. Вопрос: Сколько маленьких собак зарегистрировано для участия в выставке?

Изображение предоставлено Imgur / zakiamon

Этот вопрос взят непосредственно из домашнего задания второклассника по математике. Ой.

Ответ: 42,5 собаки.

Чтобы вычислить, сколько маленьких собак соревнуются, вы должны вычесть 36 из 49 и затем разделить полученный ответ, 13, на 2, чтобы получить 6.5 собак, или количество соревнующихся крупных собак. Но вы еще не закончили! Затем вам нужно добавить 6,5 к 36, чтобы получить количество соревнующихся маленьких собак, которое составляет 42,5. Конечно, на самом деле половина собаки не может участвовать в выставке собак, но ради этой математической задачи давайте предположим, что это так.

5. Вопрос: Найдите площадь красного треугольника.

Изображение с YouTube

Этот вопрос использовался в Китае для выявления одаренных пятиклассников. Предположительно, некоторые из умных студентов смогли решить эту проблему менее чем за одну минуту.

Ответ: 9.

Чтобы решить эту проблему, вам необходимо понять, как работает площадь параллелограмма. Если вы уже знаете, как связаны площадь параллелограмма и площадь треугольника, тогда добавление 79 и 10 и последующее вычитание 72 и 8, чтобы получить 9, должно иметь смысл, но если вы все еще не уверены, то посмотрите этот YouTube видео для более подробного объяснения.

6. Вопрос: Какова высота стола?

Изображение с YouTube

YouTube MindYourDecisions адаптировал этот ошеломляющий математический вопрос из аналогичного, найденного в домашнем задании ученика начальной школы в Китае.

Ответ: 150 см.

Изображение с YouTube

Поскольку одно измерение включает в себя рост кошки и вычитает рост черепахи, а другое дает противоположное, вы можете просто действовать так, как будто двух животных нет. Поэтому все, что вам нужно сделать, это сложить два измерения — 170 см и 130 см — и разделить их на 2, чтобы получить высоту стола 150 см.

7. Вопрос: Если стоимость биты и бейсбольного мяча вместе составляет 1,10 доллара, а бита стоит на 1 доллар больше, чем мяч, сколько стоит мяч?

Shutterstock

С математической точки зрения эта задача очень похожа на одну из других задач в этом списке.

Ответ: 0,05 доллара.

Вспомните проблему с собаками на выставке и используйте ту же логику, чтобы решить эту проблему. Все, что вам нужно сделать, это вычесть 1 доллар из 1,10 доллара и затем разделить полученный ответ, 0,10 доллара на 2, чтобы получить окончательный ответ — 0 долларов.05.

8. Вопрос: Когда у Шерил день рождения?

Изображение через Facebook / Kenneth Kong

Если у вас возникли проблемы с прочтением, см. Здесь:

«Альберт и Бернард только что подружились с Шерил, и они хотят знать, когда у нее день рождения. Черил дает им список из 10 возможных дат.

15 мая 16 мая 19 мая

17 июня 18 июня

14 июля 16 июля

14 августа 15 августа 17 августа

Затем Шерил сообщает Альберту и Бернарду отдельно месяц и день своего дня рождения соответственно.

Альберт: Я не знаю, когда у Шерил день рождения, но я знаю, что Бернард тоже не знает.

Бернард: Сначала я не знал, когда у Шерил день рождения, но теперь знаю.

Альберт: Тогда я также знаю, когда у Шерил день рождения.

Так когда у Шерил день рождения? »

Непонятно, почему Шерил не могла просто сказать Альберту и Бернарду месяц и день своего рождения, но это не имеет отношения к решению этой проблемы.

Ответ: 16 июля.

Не знаете, как найти ответ на этот вопрос? Не волнуйтесь, таково было большинство людей в мире, когда несколько лет назад этот вопрос, взятый из олимпиады по математике в Сингапуре и азиатских школах, стал вирусным.К счастью, New York Times объясняет шаг за шагом, как добраться до 16 июля, и вы можете прочитать их подробный вывод здесь.

9. Вопрос: Найдите пропущенную букву.

Изображение через Facebook / Семья Холдернесса

Это взято из домашнего задания первоклассника .

Ответ: Отсутствует буква J.

.

Когда вы складываете значения, указанные для S, B и G, сумма получается 40, и если недостающая буква J (которая имеет значение 14) делает сумму другой диагонали такой же.

10. Вопрос: Решите уравнение.

Изображение с YouTube

Эта проблема может показаться простой, но удивительное количество взрослых не могут ее решить правильно.

Ответ: 1.

Начните с решения части уравнения с делением. Для этого, если вы забыли, вам нужно перевернуть дробь и переключиться с деления на умножение, получив 3 x 3 = 9. Теперь у вас есть 9 — 9 + 1, и оттуда вы можете просто работать слева вправо и получите окончательный ответ: 1.

11. Вопрос: Где должна быть проведена линия, чтобы уравнение ниже было точным?

5 + 5 + 5 + 5 = 555.

Ответ: На знаке «+» должна быть проведена линия.

Когда вы рисуете наклонную линию в верхнем левом квадранте знака «+», она становится числом 4, и уравнение, таким образом, принимает вид 5 + 545 + 5 = 555.

12. Вопрос: Решите незаконченное уравнение.

Попытайтесь выяснить, что общего у всех уравнений.

Ответ: 4 = 256.

Формула, используемая в каждом уравнении: 4 x = Y. Итак, 4 1 = 4, 4 2 = 16, 4 3 = 64 и 4 4 = 256.

13. Вопрос: Сколько треугольников на изображении выше?

Когда Best Life впервые написал об этом обманчивом вопросе, нам пришлось попросить математика объяснить ответ!

Ответ: 18.

Некоторых людей ставят в тупик треугольники, прячущиеся внутри треугольников, а другие забывают включить гигантский треугольник, в котором находятся все остальные. В любом случае, очень немногие люди — даже учителя математики — смогли найти правильный ответ на эту проблему. А чтобы узнать о других вопросах, которые будут проверять ваше прежнее образование, ознакомьтесь с этими 30 вопросами, которые вам понадобятся для успешной сдачи 6-го класса по географии.

14. Вопрос: сложите 8,563 и 4,8292.

Сложить два десятичных знака проще, чем кажется.

Ответ: 13,3922.

Пусть вас не сбивает с толку тот факт, что у 8.563 меньше чисел, чем у 4.8292. Все, что вам нужно сделать, это добавить 0 в конец 8.563, а затем добавить, как обычно.

15. Вопрос: На озере есть участок с кувшинками. Каждый день нашивка увеличивается в размерах вдвое…

Shutterstock

… Если заплатке требуется 48 дней, чтобы покрыть все озеро, сколько времени потребуется, чтобы заплатка покрыла половину озера?

Ответ: 47 дн.

Большинство людей автоматически предполагают, что половина озера будет покрыта за половину времени, но это предположение неверно.Поскольку участок площадок удваивается в размере каждый день, озеро будет покрыто наполовину всего за день до того, как оно будет покрыто полностью.

16. Вопрос: Сколько футов в миле?

Эта задача уровня начальной школы представляет собой немного меньше решения задач и немного больше запоминания.

Ответ: 5280.

Это был один из вопросов, представленных в популярном шоу «» Вы умнее пятиклассника?

17. Вопрос: Какое значение «x» делает приведенное ниже уравнение истинным?

Shutterstock

-15 + (-5x) = 0

Ответ: -3.

Вас простят за то, что вы подумали, что ответ был 3. Однако, поскольку число рядом с x отрицательное, нам нужно, чтобы x также был отрицательным, чтобы получить 0. Следовательно, x должен быть -3.

18. Вопрос: Сколько 1,92 делится на 3?

Возможно, вам придется попросить помощи у ваших детей.

Ответ: 0,64.

Чтобы решить эту, казалось бы, простую проблему, вам нужно удалить десятичную дробь из 1,92 и действовать так, как будто ее там нет. После того, как вы разделите 192 на 3, чтобы получить 64, вы можете вернуть десятичный знак на место и получить окончательный ответ 0.64.

19. Вопрос: Решите математическое уравнение выше.

Изображение с YouTube

Не забывайте о PEMDAS!

Ответ: 9.

Используя PEMDAS (аббревиатура, указывающая порядок, в котором вы ее решаете: «скобки, показатели, умножение, деление, сложение, вычитание»), вы сначала решаете сложение внутри круглых скобок (1 + 2 = 3) и оттуда закончите уравнение, как оно написано слева направо.

20. Вопрос: Сколько всего зомби?

Чтобы найти ответ на этот последний вопрос, потребуется использовать дроби.

Ответ: 34.

Поскольку мы знаем, что на каждые три человека приходится два зомби и что 2 + 3 = 5, мы можем разделить 85 на 5, чтобы вычислить, что всего существует 17 групп людей и зомби. Затем мы можем умножить 17 на 2 и 3 и узнать, что существует 34 зомби и 51 человек соответственно. Не так уж и плохо, правда?

Чтобы узнать больше удивительных секретов о том, как прожить свою лучшую жизнь, нажмите здесь , чтобы подписаться на нас в Instagram!

Исчисление I — функции (практические задачи)

Показать уведомление для мобильных устройств

Показать все заметки Скрыть все заметки

Похоже, вы используете устройство с «узкой» шириной экрана ( i.е. вы, вероятно, пользуетесь мобильным телефоном). Из-за особенностей математики на этом сайте лучше всего просматривать в ландшафтном режиме. Если ваше устройство не находится в альбомном режиме, многие уравнения будут отображаться сбоку от вашего устройства (должна быть возможность прокручивать, чтобы увидеть их), а некоторые элементы меню будут обрезаны из-за узкой ширины экрана.

Раздел 1-1: Функции

Для задач 1–4 данные функции выполняют указанные функции оценки.2} \) Решение

  1. \ (f \ влево (4 \ вправо) \)
  2. \ (е \ влево (0 \ вправо) \)
  3. \ (f \ left ({- 3} \ right) \)
  1. \ (f \ left ({6 — t} \ right) \)
  2. \ (f \ left ({7 — 4x} \ right) \)
  3. \ (f \ left ({x + h} \ right) \)
  • \ (\ displaystyle g \ left (t \ right) = \ frac {t} {{2t + 6}} \) Решение
    1. \ (г \ влево (0 \ вправо) \)
    2. \ (g \ left ({- 3} \ right) \)
    3. \ (g \ left ({10} \ right) \)
    1. \ (g \ left ({{x ^ 2}} \ right) \)
    2. \ (g \ left ({t + h} \ right) \)
    3. \ (g \ left ({{t ^ 2} — 3t + 1} \ right) \)
  • \ (h \ left (z \ right) = \ sqrt {1 — {z ^ 2}} \) Решение
    1. \ (h \ влево (0 \ вправо) \)
    2. \ (h \ left ({- \ frac {1} {2}} \ right) \)
    3. \ (h \ left ({\ frac {1} {2}} \ right) \)
    1. \ (h \ left ({9z} \ right) \)
    2. \ (h \ left ({{z ^ 2} — 2z} \ right) \)
    3. \ (h \ left ({z + k} \ right) \)
  • \ (\ displaystyle R \ left (x \ right) = \ sqrt {3 + x} — \ frac {4} {{x + 1}} \) Решение
    1. \ (R \ влево (0 \ вправо) \)
    2. \ (R \ влево (6 \ вправо) \)
    3. \ (R \ left ({- 9} \ right) \)
    1. \ (R \ влево ({x + 1} \ вправо) \)
    2. \ (R \ влево ({{x ^ 4} — 3} \ right) \)
    3. \ (R \ left ({\ frac {1} {x} — 1} \ right) \)
  • Коэффициент разности функции \ (f \ left (x \ right) \) определяется как

    \ [\ frac {{f \ left ({x + h} \ right) — f \ left (x \ right)}} {h} \]

    Для задач 5–9 вычислить разностное отношение данной функции.

    Добавить комментарий

    Ваш адрес email не будет опубликован. Обязательные поля помечены *